You are on page 1of 90

Clinical

Pharmacology
&
Therapeutics

Alasdair Scott
BSc (Hons) MBBS PhD

2012
dr.aj.scott@gmail.com
Table of Contents
Pharmacologic Principles ......................................................................................................... 3 
Gastrointestinal ...................................................................................................................... 12 
Cardiovascular ....................................................................................................................... 18 
Respiratory ............................................................................................................................ 33 
Central Nervous System ........................................................................................................ 37 
Infection ................................................................................................................................. 51 
Endocrine ............................................................................................................................... 61 
Malignancy and Immunosuppression..................................................................................... 70 
Musculoskeletal ..................................................................................................................... 74 
Emergencies .......................................................................................................................... 80 
Revision ................................................................................................................................. 85 
Pharmacologic Principles
Contents
Pharmacokinetics .......................................................................................................................................................................... 4 
Drug Metabolism and Elimination.................................................................................................................................................. 4 
Adverse Drug Reactions ............................................................................................................................................................... 5 
Important ADRs ............................................................................................................................................................................. 6 
Side Effect Profiles ........................................................................................................................................................................ 7 
Drug Interactions ........................................................................................................................................................................... 8 
Prescribing in the Young, the Old and the Pregnant ..................................................................................................................... 9 
Prescribing in Renal Disease ...................................................................................................................................................... 10 
Prescribing in Liver Disease ........................................................................................................................................................ 11 

© Alasdair Scott, 2012 3


Pharmacokinetics
Definitions Drug Metabolism and Elimination
 Pharmacokinetics: what the body does to the drug
 Pharmacodynamics: what the drug does to the body First Pass Metabolism
 Metabolism and inactivation of a drug before it reaches
Key Concepts the systemic circulation.
 i.e. pre-systemic elimination
Clearance  Occurs in gut wall and liver
 Volume of plasma cleared of a drug per unit time  E.g. propranolol, verapamil, morphine, nitrates

Half-life Pathways of Drug Metabolism and Elimination


 Time taken for drug concentration to decline to half its  Excrete unchanged by the kidney (e.g. frusemide)
original value.  Phase 1 metabolism then renal excretion
 Depends on volume of distribution and clearance  Phase 2 metabolism then renal excretion

Volume of Distribution Phase 1 Metabolism


 Volume into which a drug appears to distribute.  Creation of reactive, polar functional groups
 High for lipid-soluble drugs  Oxidation: usually by CyP450 system
 Low for water soluble drugs  Reduction and hydrolysis

First Order Kinetics Phase 2 Metabolism


 Clearance of drug is always proportional to plasma  Production of polar compounds for renal elimination
concentration.  Either the drug or its phase 1 metabolite
 Most drugs are in this category  Conjugation reactions
 Glucuronidation, sulfonation, acetylation, methyl
Zero Order Kinetics
 Clearance of drug not always proportional to plasma Elimination: mainly renal (depends on GFR)
concentration.
 Saturation of metabolism → constant rate of elimination Cytochrome P450 System
regardless of plasma levels.  Most important system of phase 1 metabolism
 E.g. phenytoin, salicylates, ethanol  > 11 subtypes
 CyP3A4
Bioavailability  Most important subtype
 Percentage of the dose of a drug which reaches the  ≥ 30% of drugs: CCBs, β-B, statins, benzos
systemic circulation.  CyP2D6
 100% for IV administration  Second most important
 ≥20% of drugs: antidepressants, some β-B,
opiates
Multiple Dosing
 If a drug given at intervals the concentration will reach a Pro-Drugs
steady state in ~ 5 half-lives.  L-Dopa → dopamine
 Loading dose: ↓ time needed to reach a steady state.  Enalapril → enalaprilat
 Useful if long or short t½  Ezetimibe → ez-glucuronide
 Phenytoin, digoxin, amiodarone, theophylline  Methyldopa → α-methylnorepinephrine
 Azathioprine → 6-mercaptopurine (by XO)
 Carbimazole → methimazole
Therapeutic Drug Monitoring  Cyclophosphamide

Indications Pharmacogenomics
 Ix lack of drug efficacy or possibility of poor compliance  Genetically determined variation in drug response
 Suspected toxicity
 Prevention of toxicity Acetylation
 Fast vs. slow acetylators (↑↑ fast in Japan vs. Europe)
Examples  Affects: isoniazid, hydralazine and dapsone
 Aminoglycosides (essential)
 Vancomycin (essential) Oxidation
 Li (essential)  There are genetic polymorphisms for all known CyP
 Phenytoin 450 enzymes except for CyP3A4
 Carbamazepine
 Digoxin G6PD Deficiency
 Ciclosporin  Oxidative stress → haemolysis
 Theophylline  Quinolones, primaquine, nitrofurantoin, dapsone

NB. Warfarin is not monitored per se, it’s the biological effect Acute Intermittent Porphyria
which is monitored rather than the plasma drug level.  AD, ↑ in White South Africans
© Alasdair Scott, 2012
 Large no. of drugs can → attacks: e.g. EtOH, NSAIDs…
4
Adverse Drug Reactions
Classification Allergies

Type A Type 1: anaphylaxis


 Common, predictable reactions  Penicillins, contrast media
 Dose-related (but may occur @ therapeutic doses)
 Consequence of known pharmacology of the drug Type 2: cytotoxic antibodies
 E.g. causing haemolysis
Type B  Penicillins, cephalosporins, oral hypoglycaemics
 Rare, idiosyncratic reactions  Methyldopa
 Usually not dose-related
 E.g. allergies and pharmacogenetic variations Type 3: immune complexes
 Serum sickness-like reaction
Long-Term ADR  Penicillins, sulphonamides
 Dependence, addiction
 Withdrawal phenomena Type 4: cell-mediated
 Adaptive changes: e.g. tardive dyskinesia  Contact dermatitis
 Topical abx
Delayed ADR  Antihistamine cream
 Cacinogenesis
 Teratogenesis
Pseudoallergies
 Pharmacological (not immune)
Determinants
NSAIDs → bronchospasm
Drug  Shift metabolism from prostaglandins → leukotrienes →
 Pharmocodynamics bronchoconstriction
 Phamacokinetics  May occur in non-asthmatic populations
 Dose  Commoner if asthmatic
 Formulation
 Route of administration ACEi → cough and angioedema (anaphylactoid)
 Rate of aminophylline  ACEi inhibit bradykin metabolism

Patient
 Age Long-term ADRs
 Co-morbidities
 Renal: digoxin, aminoglycosides Withdrawal
 Hepatic: warfarin, opiates  Opiates
 Organ dysfunction  Benzos
 Genetic predisposition  Corticosteroids

Rebound: Worse on withdrawing the drug then before starting


 Clonidine
 β-B
 Corticosteroids

“Adaptive”
 Neuroleptics: tardive dyskinesia

Delayed ADRs
Oestrogens
 Endometrial Ca
 Breast Ca

Cytotoxics
 Leukaemia

© Alasdair Scott, 2012 5


Important ADRs
Rashes Peripheral Neuropathy
 INH
Urticaria:  Vincristine
 Immune: penicillins, cephalosporins  Amiodarone
 Non-immune: contrast, opiates, NSAIDs  Nitrofurantoin
 Penicillamine
EM: sufonamides NSAIDs, allopurinol, phenytoin, penicillin

EN: sulphonamides, OCP


Pulmonary Fibrosis
Photosensitivity: amiodarone, thiazides, sulfonylureas  Bleomycin
 Busulfan
Fixed eruptions: erythromycin, sulphonamides  Amiodarone
 Nitrofurantoin
Lupus-like reactions: hydralazine, isoniazid, penicillamine  Sulfasalazine
 Methotrexate
 Methysergide
Hepatotoxicity
Cholestatic Gynaecomastia
 Clavulanic acid: may be delayed  Spironolactone
 Fluclox: may be delayed  Digoxin
 Erythromycin  Verapamil
 Sulfonylureas (glibenclamide)  Cimetidine
 OCP  Metronidazole
 Tricyclics
 Chlorpromazine, prochlorperazine
SIADH
Hepatocellular Damage
 Carbemezapine
 Paracetamol
 Cyclophosphamide
 Valproate, phenytoin, CBZ
 Chlorpropamide
 RMP, INH, PZA
 SSRIs
 Halothane
 TCAs
 Methotrexate
 Statins

Chronic Hepatitis Gingival Hypertrophy


 INH  Nifedipine
 Methyldopa  Phenytoin
 Methotrexate  Ciclosporin

Gallstones
 OCP ↑QTc
 Fluoroquinolones: cipro
 Venlafaxine
 Neuroleptics: phenothiazines, haldol
Bone Marrow Toxicity
 Macrolides
Pancytopenia / aplastic anaemia  Anti-arrhythmics 1a/III: quinidine, amiodarone, sotalol
 Cytotoxics  TCAs
 Phenytoin  Histamine antagonists
 Chloramphenicol
 Penicillamine
 Phenothiazines
 Methyldopa

Neutropenia
 Carbamazapine
 Carbimazole
 Clozapine
 Sulfasalazine

Thrombocytopenia
 Valproate
 Salicylates
 Chloroquine
© Alasdair Scott, 2012 6
Side Effect Profiles
Cholinoceptors EPSEs
Cholinergic Anti-muscarinic Causes
 Salivation  Constipation  Typical antipsychotics
 Bronchoconstriction  Urinary retention  Rarely: metoclopramide, prochlorperazine
 Lacrimation  Mydriasis  Esp. in young women
 Urination  Blurred vision  Dyskinesias and dystonias are common c̄ anti-
 Diarrhoea  Bronchodilatation parkinsonian drugs.
 GI upset  Drowsiness
 Emesis  Dry eyes / skin Mechanism
 Miosis  D2 block in the nigrostriatal pathway
 Sweating  Excess AChM (hence effect of anti-AChM)

Causes Causes
 Anti-cholinesterases  Ipratropium Parkinsonian
 Anti-histamines  Occurs w/i months
 TCAs  Commoner in the elderly
 Antipsychotics  Bradykinesia tremor, rigidity
 Procyclidine  Rx: procyclidine (anti-AChM)
 Atropine
Acute Dystonia
 Occurs w/i hrs-days of starting drugs
Dopamine  Commoner in young males
 Involuntary sustained muscle spasm
Excess  E.g. lock jaw, spasmodic torticollis, oculogyric crisis
 Causes  Rx: procyclidine
 L-dopa
 Da agonists
 Features Akathisia
 Behaviour change  Occurs w/i days-months
 Confusion
 Subjective feeling of inner restlessness
 Psychosis
 Rx: propranolol (crosses BBB)
Deficit
 Causes
Tardive Dyskinesia
 Anti-psychotics
 Rhythmic involuntary movements of head, limbs and
 Anti-emetics: metoclopramide, prochlorperazine
trunk.
 Features
 Chewing, grimacing
 EPSEs
 ↑ PRL  Protruding, darting tongue
 Neuroleptic malignant syndrome  Occur in 20% of those on long-term neuroleptics (yrs)
 Rx
 Switch → atypical neuroleptic
 Clozapine may help
Cerebellum
 (procyclidine worsens symptoms)
 Dysdiadochokinesis, dysmetria, rebound
 Ataxia
 Nystagmus Neuroleptic Malignant Syndrome
 Intention tremor  4-10d after initiation or change of dose
 Slurred speech  Mostly in young males
 Hypotonia  Features
 Motor: severe muscular rigidity
 Mental: fluctuating consciousness
Causes  Autonomic: hyperthermia, ↑HR, sweating, ↑/↓BP
 EtOH  Blood: ↑CK, leukocytosis
 Phenytoin  Rx
 Dantrolene: inhibits muscle Ca release
 Bromocriptine / apomorphine: reverse Da block
 Cool pt.

© Alasdair Scott, 2012 7


Drug Interactions
Pharmaceutical P450 Inducers
 Take place outside the body  Phenytoin
 Mainly IV drugs being mixed together  Carbamazepine
 E.g. calcium + sodium bicarbonate → precipitation  Barbiturates
 Rifampicin
 Alcohol (chronic)
Pharmacokinetic  Griseofulvin
 St. John’s Wart
Altered Absorption
 Tetracyclines and quinolones c̄ Ca, Fe, Al
 Drugs chelate the metals and are not absorbed P450 Inhibitors
 Valproate
 INH
Displacement from Plasma Proteins  Protease inhibitors
 Warfarin + some NSAIDs  Cipro, Cimetidine
 Often clinically insignificant as clearance ↑s  Erythromycin + clarithromycin
proportionally c̄ displacement  Omeprazole
 Grapefruit juice
 Fluconazole / Fluoxetine
Metabolism
 Enzyme Inhibitors
 P450 Important Drugs Metabolised by P450
 XO: allopurinol  Ciclosporin
 Dopa decarboxylase: carbidopa  OCP
 Acetaldehyde dehydrogenase: disulfiram,
 Warfarin
metronidazole
 Epileptic drugs: phenytoin, CBZ
 Enzyme Inducers
 Statins
 Theophylline
Excretion
 Diuretics → ↓ Li clearance
 Loop diuretics: ↑ aminoglycoside ototoxicity
Warfarin
W+ W-
Indirect Interactions Enzyme inhibitors Enzyme inducers
 Diuretics and steroids → ↑ risk of digoxin toxicity EtOH
 Via ↓ K+ Simvastatin
 NSAIDs + warfarin → ↑ risk of GI bleed NSAIDs
 Abx + warfarin → ↑ bleeding risk Dipyridamole
 Abx kill GI microflora that make vit K Amiodarone

Diuretics
 Potentiate: ACEi, Li, Digoxin
 Loop → ↑ risk of ototoxicity c̄ aminoglycosides
 K-sparing → ↑ risk of hyperkalaemia c̄ ACEi

© Alasdair Scott, 2012 8


Prescribing in the Young, the Old and the Pregnant
The Elderly Teratogens
Pharmacokinetics Mechanisms
 Distribution  Orally active = crosses placenta
 ↓ body water → ↑ [water soluble drugs]  Implantation (5-17d) → abortion
 ↑ body fat → ↓ [fat soluble drugs]  Embryonic (17-57d) → structural defects
 ↓ albumin → ↑ [protein-bound drugs]  Fetogenic (maturation) → relatively less dangerous
 ↓ wt.  standard dose → ↑ [drug]
 Metabolism Common Drugs
 ↓ oxidation  ACEi: affect kidney growth
 ↓ first-pass metabolism (e.g. propranolol)  AEDs: NTDs
 ↓ induction of liver enzymes  Li: Ebstein’s anomaly
  ↑ age → ↑ t½ of hepatically metabolised drugs  Anti-folate: e.g. trimethoprim → NTDs
 E.g. Warfarin  Tetracyclines: stain teeth
 Elimination  Warfarin: cardiac defects, ↓IQ, saddle nose, blindness
 ↓ GFR  Statins
 ↓ tubular secretion

Bottom Line
 ↑ age tends to → greater and longer drug effects
Drugs to Avoid in Late Pregnancy
 Aspirin: haemorrhage, kernicterus
Altered Organ Sensitivity
 Aminoglycosides: CN8 damage
 ANS
 Anti-thyroid: goitre, hypothyroidism
 Defective compensatory mechanisms
 ↓ β-receptor density  Benzos: floppy baby syndrome
  ↓ effectiveness of drugs targeting them  Chloramphenicol: grey baby syndrome
 CNS  Warfarin: haemorrhage
 ↑ sensitivity to anxiolytics and hypnotics  Sulphonylureas: kernicterus
 ↓ Cardiac Function
 ↓ perfusion of liver and kidneys → ↓ function → ↓
metabolism or elimination of drug
Mx in Pregnancy
Compliance Problems
 Confusion ± changes in tablet morphology Hypertension
 ↓ vision  NB. Don’t prescribe ACEi to fertile young women.
 Arthritic hands  Labetalol
 Living alone  Methyldopa
 Polypharmacy  Nifedipine
 Hydralazine
Major Problem Drugs
 Affecting the Cardiovascular System DM
 Anti-hypertensives  Poor glucose control → ↑ congenital abnormalities
 Digoxin  Use insulin and/or metformin
 Diuretics
 Affecting the CNS Epilepsy
 Anti-depressants  Folic acid pre-conception
 Ant-parkinsonian  Drug levels tend to fall in pregnancy
 Hypnotics  ↑ risk of malformations (6% vs. 2%)
 ↑ risk of haemorrhagic disease of the newborn (↓K)
Neonates  Avoid valproate
 Use lamotrigine (or CBZ)
Pharmacokinetics
 Absorption: ↓ gastric motility Anticoagulation
 Distribution st
 1 trimester: LMWH
 Immature BBB  2nd trimester – 36wks: LMWH or warfarin
 ↑ body water → ↓ [water soluble drugs]  36wks – term: LMWH
 ↓ body fat → ↑ [fat soluble drugs]
 Term onwards: warfarin
 ↓ albumin → ↑ [protein-bound drugs]
 Metabolism
 ↓ P450 activity
 ↓ conjugation Drugs and Breast-Feeding
 Excretion  Sedatives (benzos, EtOH) → drowsiness
 ↓ GFR and ↓ tubular secretion  Anti-thyroid → goitre
 Tolbutamide → hypoglycaemia in infant
Bottom Line
 ↓ age tends to → greater and longer drug effects
© Alasdair Scott, 2012 9
Prescribing in Renal Disease
Important Drugs Affected by Renal Impairment Nephrotoxicity
Digoxin Gentamicin
 T½: 36 → 90hrs  Renal tubular damage
 Low therapeutic index: should be monitored  → Accumulation → ↑ nephro-/oto-toxicity
 Toxicity  Must monitor drug levels
 Nausea, xanthopsia, gynaecomastia
 A / V tachyarrhythmias, heart block Li
 Inhibits Mg-dependant enzymes
Gentamicin  e.g. adenylate cyslase
 T½: 2.5 → >50hrs  ADH requires adenylate cyclase  Li →
 Must be monitored nephrogenic DI
 Toxicity  Also causes direct tubular damage
 Ototoxic: hearing + vestibular  Must monitor drug levels
 Nephrotoxic tubules
 ↑ risk of toxicity if ↓ Na (e.g. diuretics) or dehydration Ciclosporin
 ↓ GFR: reversible
Atenolol  Damages renal tubules: irreversible
 T½: 6 → 100hrs  P450 substrate
 Toxicity  Consider monitoring
 Bradycardia, hypotension
 Worsening of PVD and heart failure ACEi / ARB
 Confusion  ↓GFR: inhibit efferent arteriorlar vasoconstriction
 CI  May be profound in RAS or CoA
 Asthma / bronchospasm
 Severe heart failure NSAIDs
 PVD  ↓ GFR: prevent afferent arteriolar vasodilatation
 Papillary necrosis
Amoxicillin
 T½: 2 → 14hrs
 Toxicity
 Seizures (in meningitis: impaired BBB)
 Rashes

Captopril
 T½: 2 → 14hrs
 Toxicity
 Hypotension
 Taste disturbance
 Cough
 ↓ GFR
 Angioedema

Vitamin D
Forms
 Colecalciferol / D3: formed in skin and found in food
 Ergocalciferol / D2: produced by UV light in fungi from
ergosterol
 Calcifediol: 25 OH-Vit D3 produced by the liver
 Calcitriol: 1, 25, (OH)2 Vit D3, produced by the kidney
 Alfacalcidol: 1 OH-Vit D3

Pathophysiology
 25 OH-Vit D3 undergoes renal 1α-hydroxylation to the
active form: 1, 25, (OH)2 Vit D3
 Impaired 1α-hydroxylase function in renal disease

In Renal Impairment
 Use alfacalcidol (1α-hydroxylated)
 Calcitriol rarely used

© Alasdair Scott, 2012 10


Prescribing in Liver Disease
Cautions in Hepatic Impairment Hepatotoxicity
Hypoalbuminaemia Cholestatic
 ↑ proportion of free drug  Clavulanic acid: may be delayed
 Phenytoin, CBZ  Fluclox: may be delayed
 Prednisolone  Erythromycin
 Diazepam  Sulfonylureas (glibenclamide)
 Tolbutamide  OCP
 Tricyclics
↓ Synthesis of Clotting Factors  Chlorpromazine, prochlorperazine
 Warfarin
 Abx (kill GI microflora) Hepatocellular Damage
 Paracetamol
↓ First Pass Metabolism  Valproate, phenytoin, CBZ
 Opiates  RMP, INH, PZA
 Phenothiazines  Halothane
 Imipramine  Methotrexate
 Statins
↓ α1-acidic Glycoprotein
 Binds basic drugs Chronic Hepatitis
 e.g. chlorpromazine, quinidine, imipramine  INH
 Methyldopa
Encephalopathy
 Sedatives / Opiates: may → coma Gallstones
 Caution c̄ drugs that may → constipation  OCP
 Anxiolytics: temazepam safest (short t½)
 Avoid chlormethiazole
 Anti-depressants: TCAs safest (but ↓ dose)
 Avoid MOAIs

Hepatorenal Syndrome
 Withdraw nephrotoxic drugs
 Modify doses of renally-excreted drugs

© Alasdair Scott, 2012 11


Gastrointestinal
Contents
Dyspepsia, GORD and PUD ....................................................................................................................................................... 13 
Laxatives ..................................................................................................................................................................................... 14 
Miscellaneous Gastrointestinal Drugs ......................................................................................................................................... 14 
GORD .......................................................................................................................................................................................... 15 
PUD ............................................................................................................................................................................................. 15 
H. pylori Eradication Therapy ...................................................................................................................................................... 15 
Ulcerative Colitis .......................................................................................................................................................................... 16 
Crohn’s Disease .......................................................................................................................................................................... 16 
C. diff Diarrhoea .......................................................................................................................................................................... 17 

© Alasdair Scott, 2012 12


Dyspepsia, GORD and PUD

Drug MOA Side Effects Contraindications Interactions Other


Mg Trisilicate Ant-acid Diarrhoea Interfere c̄ drug absorption Take when symptoms occur or are
Neutralise gastric acid - take separately expected.
Al hydroxide Ant-acid Constipation Interfere c̄ drug absorption Take when symptoms occur or are
Neutralise gastric acid - take separately expected.
Gaviscon Alginate

↓ reflux
- ↑ viscosity of stomach contents
- Form a raft on top of stomach
contents
Omeprazole PPIs GI disturbance P450 inhibitor Caution
Lansoprazole Headache - may mask symptoms of
Pantoprazole Activated in acidic pH gastric Ca
Irreversibly inhibit H+/K+ ATPase
More effective cf. H2RAs
Cimetidine H2 receptor antagonist Mainly c̄ cimetidine Cimetidine is a P450 inhibitor Caution
Ranitidine GI disturbance - may mask symptoms of
↓ gastric parietal cell H+ secretion gastric Ca
Misoprostol Prostaglandin analogue Diarrhoea is v. common Mainly used to prevent NSAID-assoc.
PUD.
Acts on parietal cells to ↓H+ secretion
Often in combination c̄ NSAID
- Diclofenac + misoprostol = Arthrotec

© Alasdair Scott, 2012 13


Laxatives

Drug MOA Side Effects Contraindications Interactions Other


Bran Bulk laxatives Bloating Bowel obstruction
Ispaghula ↑ faecal mass → ↑ peristalsis
Docusate Stimulant laxative Bowel obstruction Co-danthrusate is a mild
Glycerin (PR) ↑ intestinal motility stimulant laxative used in
Senna Rx of opioid-induced
Picosulfate constipation.
Lactulose Osmotic laxative Bowel obstruction Lactulose is used in Rx of
Macrogol ↑ stool water content hepatic encephalopathy
Phosphates (PR)
Mg Salts
Liquid paraffin Stool softener ↓ absorption of ADEK vitamins Bowel obstruction
Granulomatous reactions

Miscellaneous Gastrointestinal Drugs

Drug MOA Side Effects Contraindications Interactions Other


Hyoscine butylbromide Antimuscarinic Anti-AChM SEs Myasthenia gravis
- Buscopan - Antispasmodic - Dry mouth
- Palpitations
Mebeverine Antispasmodic
Peppermint oil
Loperamide Opioid receptor agonist Abdo cramps Infective diarrhoea
Doesn’t cross BBB  no central Colitis
effects. Caution in hepatic
impairment
Sulfasalazine 5-Aminosalicylate Sulfasalazine has ↑ SEs Caution in renal or Monitor FBC
Mesalazine - blood dyscrasias hepatic impairment
Unknown MOA - hepatitis Topical in distal disease
- rash, urticaria
- oligospermia
- pulmonary fibrosis
Budesonide Steroid Used to induce remission in
More potent cf. prednisolone ileal Crohn’s
High 1st pass metabolism  ↓
systemic effects.
Infliximab: Remicade Chimeric anti-TNF mAb Severe infections TB Screen for TB before use
Etanercept: Enbrel P75 TNFR-Fc fusion protein TB Parenteral admin
Adalimumab: Humira Human anti-TNF mAb Allergic reactions
CCF Give c̄ hydrocortisone to ↓
CNS demyelination allergic SEs
© Alasdair Scott, 2012 14
GORD PUD
Conservative Conservative
 Lose wt.  Lose wt.
 Raise head of bed  Stop smoking and ↓ EtOH
 Small regular meals ≥ 3h before bed  Avoid hot drinks and spicy food
 Stop smoking and ↓ EtOH  Stop drugs: NSAIDs, steroids
 Avoid hot drinks and spicy food
 Avoid tight clothes Medical
 Stop drugs: NSAIDs, steroids, CCBs, nitrates  OTC antacids: Gaviscon, Mg trisilicate
 H. pylori eradication: PAC500 or PMC250
Medical  Full-dose acid suppression for 1-2mo
 OTC antacids: Gaviscon, Mg trisilicate  PPIs: lansoprazole 30mg mane
 1: Full-dose PPI for 1-2mo  H2RAs: ranitidine 300mg nocte
 Lansoprazole 30mg OD  Low-dose acid suppression PRN
 2: No response → double dose PPI BD
 3: No response: add an H2RA Surgical
 Ranitidine 300mg nocte  Rarely performed
 Control: low-dose acid suppression PRN  Selective vagotomy
 Antrectomy + vagotomy
Surgical: Nissen Fundoplication  Subtotal gastrectomy + Roux-en-Y
 Indications: all 3 of:
 Severe symptoms
 Refractory to medical therapy
 Confirmed reflux (pH monitoring) H. pylori Eradication Therapy
 7 days Rx
 NB. PPIs and cimetidine → false –ve C13 breath tests
and antigen tests  stop >2wks before.

PAC 500
 PPI: lansoprazole 30mg BD
 Amoxicillin 1g BD
 Clarithromycin 500mg BD

PMC 250
 PPI: lansoprazole 30mg BD
 Metronidazole 400mg BD
 Clarithromycin 250mg BD

Failure
 95% success
 Mostly due to poor compliance
 Add bismuth
 Stools become tarry black

© Alasdair Scott, 2012 15


Ulcerative Colitis Crohn’s Disease
Acute Severe UC Acute Severe Crohn’s
 Resus: Admit, IV hydration, NBM
 Hydrocortisone: IV 100mg QDS + PR Assessment
 Transfuse if required  ↑temp, ↑HR, ↑ESR, ↑CRP, ↑WCC, ↓albumin
 Thromboprophylaxis: LMWH
 Monitoring Management
 Bloods: FBC, ESR, CRP, U+E  Resus: Admit, NBM, IV hydration
 Vitals + stool chart  Hydrocortisone: IV + PR if rectal disease
 Twice daily examination  Abx: metronidazole PO or IV
 ± AXR  Thromboprophylaxis: LMWH
 Dietician Review
Improvement → oral therapy  Elemental diet
 Switch to oral pred + 5-ASA  Consider parenteral nutrition
 Taper pred after full remission  Monitoring
 Vitals + stool chart
No Improvement → rescue therapy  Daily examination
 Discussion between pt, physician and surgeon
 Medical: ciclosporin, infliximab or visilizumab Improvement → oral therapy
 Surgical  Switch to oral pred (40mg/d)

No Improvement → rescue therapy


 Discussion between pt, physician and surgeon
Inducing Remission in Mild to Moderate Disease  Medical: methotrexate ± infliximab
 OPD treatment  Surgical
Oral Therapy
 1st line: 5-ASAs Inducing Remission in Mild to Moderate Disease
 2nd line: prednisolone  OPD treatment
 3rd line: ciclosporin or infliximab
Supportive
Topical Therapy: mainly left-sided disease  High fibre diet
 Proctitis: suppositories  Vitamin supplements
 More proximal disease: enemas or foams
 5-ASAs ± steroids (prednisolone or budesonide) Oral Therapy
 1st line
Additional Therapy: steroid sparing  Ileocaecal: budesonide
 Azathioprine  Colitis: sulfasalazine
 Infliximab: steroid-dependent pts  2nd line: prednisolone (tapering)
 3rd line: methotrexate
 4th line: infliximab or adalimumab
Maintaining Remission
Perianal Disease
 1st line: 5-ASAs PO – sulfasalazine or mesalazine
 Occurs in ~50%
 Topical Rx may be used in proctitis
 Ix: MRI + EUA
 2nd line: azathioprine or 6-mercaptopurine
 Relapsed on ASA or are steroid-dependent  Rx
 Use 6-mercaptopurine if azathioprine intolerant  Oral Abx: metronidazole
 Immunosuppression ± infliximab
 3rd line: infliximab / adalimumab
 Local surgery ± seton insertion

Maintaining Remission
Elective Surgery  1st line: azathioprine or mercaptopurine
 2nd line: methotrexate
Indications  3rd line: Infliximab / adalimumab
 Chronic symptoms despite medical therapy
 Carcinoma or high-grade dysplasia
Elective Surgery
Procedures
Indications
 Proctocolectomy c̄ end ileostomy or IPAA
 Abscess or fistula
 Total colectomy c̄ (IRA)
 Perianal disease
 Chronic ill health
 Carcinoma

Procedures
 Limited resection: e.g. ileocaecal
 Stricturoplasty
© Alasdair Scott, 2012
 Defunction distal disease c̄ temporary loop ileostomy
16
C. diff Diarrhoea
High Risk Abx
 Cephalosporins
 Clindamycin

General
 Stop causative Abx
 Avoid antidiarrhoeals and opiates
 Enteric precautions

Specific
1st line: Metronidazole 400mg TDS PO x 10-14d

Metronidazole Failure
 Vanc 125mg QDS PO x 10-14d

Severe: Vanc 125mg QDS PO 1st line (may add metro IV)
 ↑ to 250mg QDS if no response (max 500mg)
 Urgent colectomy may be needed if
 Toxic megacolon
 ↑ LDH
 Deteriorating condition

Recurrence (15-30%)
 Reinfection or residual spores
 Repeat course of metro x 10-14d
 Vanc if further relapse (25%)

Treatment Failure
 Defined as no clinical response after 1wk
 C. diff toxin assay will remain positive for ≥2wks
following original infection.

© Alasdair Scott, 2012 17


Cardiovascular
Contents
Diuretics ....................................................................................................................................................................................... 19 
The Renin-Angiotensin System ................................................................................................................................................... 20 
Beta Blockers .............................................................................................................................................................................. 21 
Miscellaneous Anti-Hypertensives .............................................................................................................................................. 22 
Calcium Channel Blockers .......................................................................................................................................................... 23 
Nitrates ........................................................................................................................................................................................ 24 
Miscellaneous Anti-Anginals ....................................................................................................................................................... 24 
Vaughan-Williams Classification ................................................................................................................................................. 25 
Anti-Arrhythmic Agents ................................................................................................................................................................ 26 
Anti-Platelet Agents ..................................................................................................................................................................... 27 
Lipid Lowering Therapy ............................................................................................................................................................... 28 
Warfarin ....................................................................................................................................................................................... 29 
Heparin ........................................................................................................................................................................................ 30 
Thrombolytics .............................................................................................................................................................................. 30 
Atrial Fibrillation ........................................................................................................................................................................... 31 
Angina Pectoris ........................................................................................................................................................................... 31 
Chronic Heart Failure .................................................................................................................................................................. 32 
Hypertension ............................................................................................................................................................................... 32 

© Alasdair Scott, 2012 18


Diuretics

Drug MOA Side Effects Contraindications Interactions Other


Frusemide Loop diuretics ↓ Na Refractory hypokalaemia ↑ toxicity of Monitor U+Es
Bumetanide ↓K Anuric renal failure - digoxin (due to ↓K)
Inhibit Na / K / Cl triple transporter in ↓ Ca - NSAIDs May add K+-sparing diuretic to ↓
ascending loop of henle ↓ Mg - Gentamicin K loss.
- ↑ NaCl excretion ↑ urate - Li
Postural hypotension
Tinnitus / deafness (rare)
Bendrofluazide Thiazide diuretics ↓Na Refractory hypokalaemia ↑ toxicity of
Metolazone ↓K Gout - digoxin
Chlortalidone Inhibit NaCl transporter in DCT ↑ Ca Severe renal failure - Li
- ↑ NaCl excretion ↑ urate
Postural hypotension
↑ glucose and DM
Spironolactone Aldosterone receptor antagonists ↑K ↑K ↑ toxicity of Spiro doses
Eplerenone - ↑ Na excretion Gynaecomastia (spiro) P - digoxin - 25mg OD for HF
- ↓ K and H excretion Addison’s - Li - 100-400mg OD for diuresis
Amiloride Block Na channels in collecting ↑K Typically used in combo c̄ K+-
Triamterine tubules GI upset wasting diuretics.
- ↑ Na excretion
- ↓ K and H excretion
Acetazolamide Carbonic anhydrase inhibitor Rash: EM → SJS Sulfonamide hypersensitivity A sulphonamide
- ↑ HCO3 excretion Peripheral tingling
Use
- open- / closed-angle glaucoma
Mannitol Osmotic diuretic Use
- ↓ IOP
- ↓ ICP

© Alasdair Scott, 2012 19


The Renin-Angiotensin System

Drug MOA Side Effects Contraindications Interactions Other


Lisinopril ACEi Hypotension Suspected or confirmed bilat ↑ risk of RF c̄ NSAIDs Monitor U+Es
Captopril - esp. c̄ diuretics, HF RAS - ↑ Cr >30% → MRA
Ramipril Inhibits conversion of ang I → ang II RF Diuretics, TCAs and
Perindopril ↑K+ Angioedema/hypersensitivity antipsychotics → risk of ↓↓BP Titrate dose
Dry cough: 10-20% to ACEi
- 2O to ↑ bradykinin Caution c̄ drugs that ↑ K+ Avoid in young women who
+
Angioedema (~0.1%) Salt substitutes (contain K ) - e.g. spiro might become pregnant
- consider β-B
P/B
↓ dose in RF
Candesartan Ang II receptor (AT1) antagonists As for ACEI but no cough P/B As above
Irbesartan
Losartan Don’t inhibit kinin breakdown  no Caution in RAS
cough

Physiology Principal Indications


 Angiotensinogen is an α2-globulin released by the liver.  Heart failure
 Renin from the JGA converts angiotensinogen → ang 1  HTN
 ACE is produced by pulmonary epithelial cells and converts ang 1 → ang 2.  Post-MI
 Ang 2 acts via AT1 receptor  Angina
 Vasoconstriction  Diabetic Nephropathy
 Sympathetic activation
 Aldosterone release from adrenal cortex
 ↑ renal Na reabsorption
 ADH release
 Ang 2 is degraded by angiotensinases in RBCs

© Alasdair Scott, 2012 20


Beta Blockers

Drug MOA Side Effects Contraindications Interactions Other


Cardioselective Block β receptors Bronchospasm Asthma / bronchospasm Verapamil and Propranolol is v. lipid
- Bisoprolol - inc. cardioselective PVD diltiazem → risk of AV soluble and easily
- Atenolol Actions via β1 → ↓ CO Severe bradycardia block and ↓HR crosses the BBB → CNS
- Metoprolol - ↓↓ heart rate ↓HR and ↓BP Severe heart failure effects (nightmares).
- Esmolol - ↓ contractility 2nd-3rd degree AV block Enhanced ↓BP effects
- Nebivolol - small ↓ BP: central effect + ↓ renin Peripheral vasoconstriction c̄ other anti-HTN drugs Atenolol is water soluble
- cold extremities and doesn’t cross BBB.
Non-selective Effects - worsened Raynaud’s/PVD Caution in DM as ↑ likelihood of Blocks symptoms of
- Carvedilol - ↑ diastolic perfusion hypoglycaemia ↓glucose c̄ insulin
- Propranolol - ↓ O2 demand Lethargy / fatigue - mask symptoms
- Sotalol - ↓ afterload Nightmares
- Labetalol ↓ dose in renal impairment
Metabolic May ↓ dose in hepatic impairment
ISA - ↓HDL
- Acebutolol - ↑ TGs
- Pindolol - ↑ risk of new onset DM (esp.
- Oxprenolol c̄ thiazides)

Vasodilating
- Carvedilol
- Labetalol
- Nebivolol

Physiology Pharmacology Principal Indications


 Angina
β1 receptors: Some β-B have arteriolar dilating effects which ↓TPR  Heart failure
 Heart: ↑ rate and contractility  Block α1 receptors  Acute MI
 Kidney: ↑ renin release from JGA  Carvedilol, labetalol, nebivolol  Arrhythmias
 HTN
β2 receptors: Cardioselective agents have ↓ β2 effects.  Long QT syndrome
 Bronchi, GIT: SM relaxation  Prophylaxis vs. variceal haemorrhage
 Skeletal muscle: arteriolar dilatation ISA: partial agonist activity at adrenoceptors  Migraine prophylaxis
 Liver + skeletal muscle: glycogenolysis and  ↓ bradycardia  Thyrotoxicosis
gluconeogenesis  ↓ cold extremities  Glaucoma
 Anxiety
β3 receptors: Lipophilic compounds more likely to → CNS effects
 Adipose tissue: lipolysis  Propranolol, metoprolol

Hydrophilic compounds may accumulate in renal


failure
 Atenolol, sotalol

© Alasdair Scott, 2012 Esmolol is v. short acting: use IV 21


Miscellaneous Anti-Hypertensives

Drug MOA Side Effects Contraindications Interactions Other


Non-Selective α-blockers Postural Hypotension B ↑s hypotensive effects of Phentolamine is short-
- Phenoxybenzamine Dizziness - diuretics acting and can be used to
- Phentolamine α1 Headache - β-B control BP in phaeo.
- systemic vasodilatation Urinary incontinence - CCBs
Alpha 1 - relaxation of internal urethral - esp. women Phenoxybenzamine is
- Doxazosin sphincter Blurred vision long-acting and is used to
- Prazosin maintain α blockade once
BP controlled.

Doxazosin and tamsulosin


are used in Rx of BPH
Clonidine Centrally-acting α2 agonist Rebound HTN on withdrawal
- ↓ CO Postural hypotension
- ↓ PVR Constipation
Nausea
Dry mouth
Methyldopa Centrally-acting α2 agonist Blood dyscrasias L Avoid w/i 2wks of MAOI Mainly used in P
Pro-drug Hepatotoxic Depression
- → α-methyl NA Drug-induced lupus
Drowsiness
Hydralazine Vasodilator Drug-induced lupus SLE Mainly used in P
- arteries > veins ↑HR
GI upset ↓ dose in hepatic or renal
Headache impairment
Nitroprusside Vasodilator Hypertensive crisis
- arteries > veins
Minoxidil Vasodilator Hypertrichosis Used to promote hair
growth

© Alasdair Scott, 2012 22


Calcium Channel Blockers

Drug MOA Side Effects Contraindications Interactions Other


Dihydropyridines Mainly Arterial SM Activity Flushing Cardiogenic shock Risk of ↓↓BP c̄ α/β-B Indications
- Nifedipine - vasodilatation (inc. coronary) Headache - Angina
- Amlodipine - particularly pre-capillary arterioles Ankle oedema Unstable angina Fx ↑d by grapefruit - Prinzmetal’s angina
- esp. amlodipine - HTN
↓TPR → ↑ sympathetic tone → ↑ HR Dizziness Significant AS Fx ↓d by: - Raynaud’s
↓ BP - Rifampicin
w/i 1mo of MI - CBZ + phenytoin
Gingival hypertrophy
- esp. nifedipine Nifedipine only
- ↑s fx of digoxin

Non-dihydropyridines Mainly Cardiac Activity Headache HF Risk of AV block, HF and Indications


- Diltiazem - -ve inotropic effect (esp. verapamil) Flushing 2nd/3rd degree AV block asystole c̄ β-blockers. - Angina
- Verapamil - verapamil also slows conduction @ AV block - HTN
SA and AV nodes HF ↑s fx of digoxin - Arrhythmias (Verap)
↓BP
Some activity @ arterial SM (<DHPs) Ankle oedema Fx of verapamil ↑d by:
Constipation - Grapefruit juice
Gynaecomastia (verap) - Macrolides

↑ risk of myopathy c̄
simvastatin

Pharmacology Indications
MOA Verapamil and Diltiazem
 Bind α1 subunit of L-type Ca channel at distinct sites  HTN
 Prevent channel opening and inhibit Ca entry  Angina
 AF
Effects
 All CCBs are vasodilators: ↓ afterload Nifedipine MR and Amlodipine
 Also dilate coronary arteries  HTN (long-acting)
 Pre-capillary vasodilatation → transudative oedema  Angina: esp. good for Prinzmetal’s
 Dihydropyridines act only @ arterial SM and can → reflex tachycardia  Raynaud’s
 Avoid short acting preparations
 Verapamil is highly negatively inotropic
 CI in HF and c̄ β-B
 Verapamil is also negatively chronotropic
 Diltiazem is less negatively inotropic and chronotropic than verapamil

© Alasdair Scott, 2012 23


Nitrates

Drug MOA Side Effects Contraindications Interactions Other


GTN NO donor c̄ rapid onset and short ↓BP (inc. postural) AS and MS Sildenafil, tadakafil and SL spray or tabs
duration (30min) Headache ↓↓BP vardenafil are CI → ↓↓BP (300ug)
Syncope Constrictive pericarditis
High 1st pass metabolism Dizziness Tamponade ↓s fx of heparins if given IV Use for relief of pain
Flushing HOCM in angina, ACS
Mainly venodilation → ↓ preload Reflex tachycardia ↓↓Hb
Small ↑ coronary vasodilatation Glaucoma (closed)
ISMN / ISDN Long-acting nitrates Hypovolaemia
↑ ICP
ISMN is active metabolite of ISDN

MN avoids unpredictable first-pass


metabolism of DN

Tolerance develops quickly: need 8h


drug-free period (usually @ night)

Miscellaneous Anti-Anginals

Drug MOA Side Effects Contraindications Interactions Other


Nicorandil K+ATP channel activator + nitrate Headache Cardiogenic shock Sildenafil → ↓↓BP Indications
component Flushing - Uncontrolled angina
Arterial and venous dilator Dizziness
GI ulcers (rare)
Ivabradine Inhibits”funny” current in SA node Visual changes ↓↓BP or ↓↓HR Subject to hepatic induction / inhibition Indication
→ ↓ pacemaker activity → ↓ HR ↓HR and HB ACS - Angina (useful if β-B CI)
Strong CYP inhibitor
Trimetazidine Inhibits fatty acid oxidation Indication
→ ↑ myocardial glucose use - Angina
Ranolazine Inhibits late Na current Indication
- Angina

© Alasdair Scott, 2012 24


Vaughan-Williams Classification
Class I Class IV
 Na+ channel blockers (local anaesthetics).  CCBs: verapamil and diltiazem (non-DHPs)
 All slow conduction  Site of action: AVN
 Subclasses have additional effects on action potential  MOA: Slow AVN conduction
 Use-dependent: preferentially block open or refractory Na+ channels  Indications: prevent recurrence of SVT, AF (rate control), acute SVT
 Caution: negative inotropes
1a: quinidine, procainamide, disopyramide
 Sites of action: A, SAN, AVN, V Unclassified
 Repolarisation: Prolonged → ↑ AP duration
 Indications: V arrhythmias Digoxin
 SEs: anti-AChM, negative inotropes  Cardiac glycoside
 MOA
1b: lignocaine, mexiletine  Positive inotrope: inhibits myocyte Na+/K+ ATPase → ↑ Na+ & ↑Ca2+
 Sites of action: V only  Negative chronotrope
 Repolarisation: Shortened → normal or ↓ AP duration  Slows AV conduction → ↓ rate and ↑ AVN refractory period
 Indications: V arrhythmia following MI  Only slows resting rate, NOT exercise rate
 Indication: SVT and AF
1c: flecainide  Caution
 Sites of action: A, SAN, AVN, V  Toxicity can → arrhythmias
 Repolarisation: Little effect  ↑ toxicity if ↓K (reduced competition for binding site)
 Indications: Pre-excited AF (WPW), Acute AF
Adenosine
 MOA
Class II  Acts @ A1 receptors in cardiac tissue → myocyte hyperpolarization
 β-Blockers: metoprolol, propranolol, esmolol, atenolol  Transient AV block
 Sites of action: A, SAN, AVN, V  Indication: SVTs
 MOA
 ↑ refractory period of AVN → slow AVN conduction
 Prevent arrhythmias due to sympathetic discharge (e.g. following MI)
 Indications: post-MI, AF (rate control), SVT Clinical Classification
 Caution: negative inotropes
AVN
 Adenosine
Class III  β-B
 Amiodarone and sotalol  Verapamil
 Sites of action: A, SAN, AVN, V  Digoxin
 MOA  Use: AF, SVT
 K+ channel blockers
 ↑↑ refractory period = ↑QTc Atria and Ventricles
 Indications: V and SV arrhythmias, pre-excited AF (WPW)  Class 1a, 1c and amiodarone
 Caution: can → arrhythmias (esp. TDP)
Ventricles only
 Class 1b

© Alasdair Scott, 2012 25


Anti-Arrhythmic Agents

Drug Class Indications Side Effects Contraindications Interactions Other


Disopyramide Class Ia Ventricular arrhythmias VT, VF, TdP Heart block (2/3)
(esp. post-MI) Anti-muscarinic
Lignocaine Class Ib Ventricular arrhythmias Drowsiness Heart block Cimetidine ↑s lidocaine levels IV use only
(esp. post-MI) Paraesthesia
Dizziness
Bradycardia → cardiac arrest
Flecainide Class Ic Pre-excited AF (WPW) Strong –ve inotrope Structural heart
Cardioversion in AF - Oedema disease
Suppress V ectopics - Dyspnoea Post-MI
Amiodarone Class III SVT Eye – corneal microdeposits Thyroid disease β-B and CCB → ↑ risk of HB Accumulates in body:
AF / flutter Sinus bradycardia - V. long t½ (10-100 days)
Pre-excited AF Thyroid – hyper / hypo ↑ QTc → TdP ↑s levels of - Extensively tissue bound
V arrhythmias (inc. VF) - digoxin (halve dig dose)  requires loading dose
Lung – pulmonary fibrosis - warfarin
- phenytoin Monitor:
GI/Liver – ↑ LFTs - TFTs, LFTs (base + 6-moly)
– N/V ↑ risk of V arrhythmias c̄ : - K+ (baseline)
- Class III/Ia antiarrhythmics - CXR (baseline)
Neuro – peripheral neuropathy - TCAs, antipsychotics
- Erythromycin Avoid sunlight
Skin – photosensitivity
– blue-grey discoloration
– phlebitis (give centrally)
Digoxin Cardiac AF / flutter Toxicity Complete heart block Dig fx/toxicity ↑d by: Caution
glycoside SVT - Any arrhythmia VF/VT - CCB (esp. verapamil) - Renal excretion caution
(HF) - e.g. SVT c̄ AV block HOCM - Amiodarone (halve dig dose) in impairment
- Nausea SVTs 2O to WPW - Diuretics (loop/thiazide due - e.g. in the elderly
- Xanthopsia to ↓K+)
- Confusion Monitor
- ↑ K+ ↓ digoxin intestinal absorption: - U+Es
- Antacids - Drug levels (6h post-dose)
Chronic - Cholestyramine
- Gynaecomastia Load then maintenance

“Reverse tick” ECG


- not a sign of toxicity
Adenosine SVT: Dx and Rx Bronchospasm Asthma Fx prolonged by dipyridamole t½ = 8-10secs
Chest pain Heart Block (2/3)
Flushing Sick sinus syndrome Fx ↓ by theophylline
Nausea
Light-headedness

© Alasdair Scott, 2012 26


Anti-Platelet Agents
Drug MOA Side Effects Contraindications Interactions Other
Aspirin Irreversible, non-selective COX inhibitor Gastritis <16yrs (Reyes syn.) ↑ risk of bleeding c̄ other Stop 7d before surgery if
→ ↓ plat TxA2 → Gastric ulceration - except in Kawasaki’s anti-coagulants and anti- sig. bleeding expected.
- ↓ plat activation Bleeding Active PUD platelets: W+
- ↓ platelet adhesion, aggregation Bronchospasm Bleeding disorders Max dose: 4g/day
Gout ↑s fx of
Relatively platelet-specific @ low dose Renal failure R (if GFR < 10ml/min) - sulphonylureas
- 75-150mg Gout P, B - methotrexate
Ototoxic in OD: tinnitus
Caution:
- Asthma
- Uncontrolled HTN
Clopidogrel Thienopyridine Bleeding Severe L Avoid c̄ warfarin. Prodrug converted by
(Plavix) - esp. GI or intracranial B hepatic CYP enzymes.
Irreversible adenosine R antagonist GI upset
- inhibits ADP-induced fibrinogen binding Dyspepsia / PU Used following bare-
to GPIIb/IIIa TTP (rare) metal or drug eluting
Blood dyscrasias (rare) stents.

Stop 7d before surgery if


sig. bleeding expected
Abciximab MAb or synthetic inhibitors of GPIIb/IIIa Bleeding Many, mainly concerned c̄ Only abciximab can be
Tirofiban Thrombocytopenia bleeding. given PO
Eptifibatide
Given to high-risk pts. c̄
NSTEMI
Dipyridamole Phosphodiesterase inhibitor Headache Myasthenia gravis Enhances effects of May be used c̄ aspirin in
- ↑cAMP inhibits plat aggregation adenosine 2O prevention of stroke
TxA2 synthetase inhibitor

Indications for Anti-Platelet Therapy Recommendations Aspirin OD


 ACS  Clopidogrel is preferred over aspirin + DP-MR for 2O  Mixed met acidosis and resp alkalosis
 Secondary prevention prevention after stroke  Pres: vomiting, dehydration, tinnitus, hyperventilation
 IHD  Aspirin + DP-MR is preferred over clopi following a TIA.  Ix
 Stroke  Clopidogrel + aspirin is given in the emergent Mx of  Salicylate and paracetamol level
 TIA STEMI and NSTEMI  U+E, ABG, glucose, LFTs, INR.
 PVD  Continue clopi for 12mo in NSTEMI or 1mo in  Rx
 Primary prevention STEMI  Gastric lavage if <1h since OD
 10yr CVD risk >20% + controlled BP  Clopi + aspirin not beneficial following stroke  Correct dehydration and acidosis (c̄ HCO3-).
 Tissue heart valve replacements  Alkalinize urine: NaHCO3 + KCl
 Haemodialysis: levels >700mg/L, cardiac/renal
failure, seizures

© Alasdair Scott, 2012 27


Lipid Lowering Therapy

Drug MOA Side Effects Contraindications Interactions Other


Rosuvastatin Statins Myositis L ↑ risk of myositis c̄ : Monitor
Atorvastatin HMG-CoA Reductase Inhibitors – block - stop if CK 5x ULN P (contraception needed - Fibrates - LFTs ± CK
Simvastatin rate-limiting step in cholesterol synthesis - can → rhabdo + ATN during use and for 1mo after) - Macrolides
Lovastatin - Azoles Take statins nocte
Pravastatin → ↓ hepatocyte cholesterol → ↑ hepatic Deranged LFTs - Grapefruit juice (ALS) - ↑ cholesterol synth
LDL receptors → ↓LDL cholesterol GI upset - Protease Inhibitors overnight
(↓ing potency) - Ciclosporin
↑ HDL - Nicotinic Acid
↓ TGs (mild)
Mild W+
Bezafibrate Fibrates Gallstones Gallbladder disease ↑ risk of myositis c̄ statins Use
Gemfibrozil Stimulate lipoprotein lipase → GI upset PBC ↑s fx of anti-diabetics - hypertriglyceridaemia
- ↓TG: 36% ↓ appetite ↓ albumin
- ↓LDL: 8% Myositis - (esp. nephrotic syndrome) W+
- ↑HDL: 10% Blood dyscrasias RLPB
Cholestyramine Anion Exchange Resin GI upset Complete biliary obstruction ↓s absorption of other Don’t take w/i ~3h of
Cholestipol Binds bile acids and prevents - bloating drugs: e.g. digoxin other drugs.
enterohepatic recycling - constipation
 liver must synthesise more bile acids - N/V
from cholesterol → ↑ LDL receptors… Can ↑TGs
Impair absorption of fat
sol drugs / vits (ADEK)
Nicotinic Acid Inhibits cholesterol and TG synthesis Flushing Peptic ulcer
↑ HDL n/v
Ezetimibe Inhibits intestinal cholesterol absorption GI upset ↑ risk of myositis c̄ statins
Pro-drug Myalgia
Omega-3 FAs ↓ TGs GI upset
Orlistat Pancreatic lipase inhibitor GI upset Cholestasis Warfarin Use
→ impaired absorption of dietary fat - steatorrhoea - Difficulty controlling INR - adjunct in obesity Mx
- abdo distension
↓ absorption of fat-sol
drugs and vitamins

Statin Indications
 Any known CVD
 DM (age >40)
 10yr CVD risk ≥20%
 Aim: TC ≤4mM

© Alasdair Scott, 2012 28


Warfarin
MOA INR Target and Duration
 Inhibits Vit K epoxide reductase
 Prevents recycling of Vit K → functional Vit K deficiency Indication INR Duration
 Inhibits synthesis of factors 2, 7, 9, 10, C and S DVT prophylaxis 2-2.5
 Initially procoagulant: protein S is depleted first. Calf DVT 2.5 Cause known: 6wks
No cause: 3mo
Indications Above knee DVT 2.5 Cause known: 3mo
No cause: 6mo
Treatment Prophylaxis PE 2.5 Cause known: 3mo
No cause: 6mo
 VTE  VTE
Recurrent DVT/PE 2.5 Indefinite
 AF
3.5 if on W
 Mechanical heart valves
AF 2.5 Indefinite
 Large anterior MI (for 3mo)
Mitral valve 2.5 Indefinite
 Dilated cardiomyopathy / LV aneurysm disease
NB. Pts. c̄ Ca-assoc. VTE should initially be treated for 6mo Anti-phospholipid 3.5 Indefinite
with therapeutic dose of LMWH rather than warfarin. syn.
Metal valves 3.5 Indefinite
Pharmacokinetics
Dosing: Tait Model
 Long t½: 40hrs
 Day 1-4: warfarin 5mg OD @ 6pm
 Takes 16hrs to affect INR
 Day 5: check INR and adjust dose according to table
 Peak INR effect of a dose seen @ 2-3d
 Day 8: check INR and adjust dose according to table
 Effect of a given dose lasts 4-5d
 >Day 8: check INR every 4 days and dose accordingly
 Highly albumin-bound
 OPD: warfarinisation over 3-4wks is safe and effective
 CyP metabolism

Side Effects Raised INR


 Haemorrhage, bruising
Major bleeding
 Skin necrosis (due to protein S deficiency)
 Stop warfarin
 Purple toe syndrome (cholesterol embolism)
 Vit K IV
 Osteoporosis
 Prothrombin complex concentrate
 Hepatic dysfunction  FFP 15ml/kg if unavailable

Caution >8 (no bleed or minor bleeding)


 Hepatic impairment: avoid if severe  Stop warfarin and restart when INR <5
 Renal impairment: avoid if severe  Vit K IV/PO if risk factors for bleeding
 Alcoholics  E.g. head injury, stroke, epistaxis, prev Hx

Contraindications 6 – 8 (no bleed or minor bleeding)


 Pregnancy  Stop warfarin and restart when INR <5
 Teratogenic in 1st trimester
 Foetal haemorrhage in 3rd trimester 4.5 – 6
 PUD  ↓ or omit warfarin and restart when INR <5
 Severe HTN
 Caution if R/L, recent surgery, risk of falls Vitamin K
 Onset of action = 6h
Interactions  Oral is as efficacious as IV
 Oral Vit K can → prolonged anticoagulant resistance
 If continuing anticoagulation, avoid if possible
W+ W-
 Continuing warfarin: 0.5mg slow IV
Enzyme inhibitors CBZ
 Discontinuing warfarin: 2.5-5mg IV
EtOH Rifampicin
Simvastatin OCP PCC
NSAIDs Phenytoin  Factors 2, 7, 9 and 10
Dipyridamole Barbiturates
 Immediate reversal of anticoagulation
Amiodarone St. John’s Wart
 Temporary effect: give c̄ Vit K
Abx (may also ↓)
 Risk of VTE and v. expensive
Cranberry juice
VTE Prophylaxis on Long-Haul Flights
 Low risk: avoid dehydration, regularly flex ankles
 Mod risk: as above + compression travel socks
 Previous VTE, GA w/i last 1-2mo
 High risk: as above + consider LMWH before flight
 Surgery under GA w/i last 1mo
© Alasdair Scott, 2012 29
Heparin Thrombolytics
MOA MOA Indications
 Co-factor for ATIII: inhibits factors 2, 10, 11 and 12.  Convert plasminogen → plasmin  STEMI
 LMWH and fondaparinux only inhibit factor 10.  Plasmin breaks down fibrin  Stroke
 Life-threatening PE
LMWH or UH  Acute limb ischaemia
 LMWH has longer t½ and response is more predictable  no monitoring needed
 UH has rapid onset and short t½ Streptokinase (derived from streptococci)
 Useful when rapid control over effects needed (e.g. risk of bleeding)
 Less risk of HIT and osteoporosis c̄ LMWH Side Effects
 Bleeding
Indications  Allergic reactions
 Rash
 Anaphylaxis
Treatment Prophylaxis
 Reperfusion dysrhythmias after MI
 VTE  VTE
 Hypotension
 ACS  Coagulation in extra-corporeal circuits
 Acute arterial obstruction  Development of Abs: only use streptokinase once

Side effects Admin: infusion over 1h


 Thrombocytopenia
 Immune-mediated
 Develops ~6d after initiation (2-3% of people c̄ UH) Rh-TPA
 → thrombosis
 Osteoporosis (long-term use) Side Effects
 Hyperkalaemia: heparin inhibits aldosterone  Bleeding, hypotension, reperfusion dysrhythmias

Contraindications Admin
 Tenectaplase, reteplase: bolus
 Bleeding disorders  Cerebral haemorrhage
 Alteplase: infusion
 Plats <60  Severe HTN
 Give c̄ UH heparin IV for 24-48 to avoid rebound hypercoaguable state.
 Previous HIT  Neurosurgery
 PU

Contraindications
Dosing
 LMWH: e.g. enoxaparin Absolute Relative
 Prophylaxis: 20-40mg pre- and post-surgery Haemorrhagic stroke at any time TIA in last 6mo
 Treatment: 1.5mg/kg/24h Ischaemic stroke in last 6mo Warfarin
 UH CNS trauma or neoplasms Pregnancy or w/i 1wk post-partum
 5000iu bolus IV over 30min Major trauma/surgery in last 3wks Refractory resuscitation
 Infuse UF @ 18iu/kg/h GI bleed w/i last 1mo Refractory HTN (>180/110)
 Check APTT @ 6h (aim for 1.5-2.5x control) Known bleeding disorders Advanced liver disease
Aortic dissection Infective endocarditis
Non-compressible puncture (e.g. LP) Acute peptic ulcer

© Alasdair Scott, 2012 30


Atrial Fibrillation Angina Pectoris
Acute (<48h) Lifestyle
 Unstable → emergency cardioversion  Stop smoking
 Control ventricular rate: β-B or CCB  Wt. loss and ↑ exercise
 Start LMWH  Healthy diet: oily fish, fruit, veg, ↓ sat fats, ↓ Na
 Cardiovert: electrical or flecainide / amiodarone

Paroxysmal AF Medical
 Self-limiting, <7d, recurs
 Anticoagulate: use CHADSVAS 2O Prevention: prevent cardiovascular events
 Rx “pill-in-pocket” : flecainide, propafenone  Aspirin 75mg OD
 Prevention: β-B, sotalol or amiodarone  ACEi (esp. if angina + DM)
 Statins: simvastatin 40mg
Persistent AF  Antihypertensives
 >7d, may recur even after cardioversion
Anti-anginals: prevents angina episodes
Try rhythm control first-line if 1. GTN (spray or SL) + either
st
 Symptomatic or CCF  1 : β-B (e.g. Atenolol 50-100mg OD)
 Younger (<65)  2nd: CCB (e.g. Verapamil 80mg TDS)
 Presenting first time c̄ lone AF 2. If either β-B or CCB doesn’t control symptoms, try the
other option.
 Secondary to treated precipitant
3. Can try β-B + dihydropyridine CCB
 e.g. amlodipine 10mg/24h
Rhythm Control
4. If symptoms still not controlled
 TTE first: structural abnormalities
 ISMN 20-40mg BD (8h washout @ PM) or slow-
 Anticoagulate c̄ warfarin for ≥ 3wks
release nitrate (Imdur 60mg OD)
 or use TOE to exclude intracardiac thrombus.
 Ivabradine (esp. if can’t take β-B)
 Pre-Rx ≥4wks c̄ sotalol or amiodarone if ↑ risk of failure
 Nicorandil 10-30mg BD
 Electrical or pharmacological cardioversion  Ranolazine
 ≥4 wks anticoagulation afterwards (target INR 2.5)

Maintenance antiarrhythmic
Interventional: PCI
 Not needed if successfully treated precipitant
st
 1 : β-B (e.g. bisoprolol, metoprolol). Indications
 2nd: amiodarone  Poor response to medical Rx
 Refractory angina but not suitable for CABG
Rate control (target <90bpm at rest):
 1st line: β-B or rate-limiting CCB (NOT both!) Complications
 2nd line: add digoxin (don’t use as monotherapy)  Restenosis (20-30% @6mo)
 3rd line: consider amiodarone  Emergency CABG (<2%)
 MI (<2%)
Mx of Permanent AF  Death (<0.5%)
 Failed cardioversion / unlikely to succeed
 AF >1yr, valve disease, poor LV function Clopidogrel ↓s risk of restenosis
 Pt. doesn’t want cardioversion  Bare metal stent: 1mo
 → Rate control  Drug-eluting (e.g. sirolimus) stent: 1yr

CHA2-DS2-VAS Score
 Determines necessity of anticoagulation in AF Surgical
 Warfarin CI in AF  CABG
 Bleeding diathesis, ↓plats, BP > 160/90, poor
compliance
 Dabigatran may be cost-effective alternative.

CHA2-DS2 VAS
 CCF  Vascular disease
 HTN  Age: 65-74yrs
 Age ≥75 (2 points)  Sex: female
 DM
 Stroke or TIA (2 points)

Score
 0: aspirin 300mg
 ≥1: Warfarin

© Alasdair Scott, 2012 31


Chronic Heart Failure Hypertension
General Mx Do ABPM to confirm Dx before Rx (unless severe HTN)

1O/2O Cardiovascular Risk Lifestyle interventions


 Stop smoking  ↑ exercise
 ↓ salt intake  ↓ smoking, ↓ EtOH, ↓ salt, ↓ caffeine
 Optimise wt.: ↑ or ↓ - dietician
 Supervised group exercised based rehab programme Indications for Pharmacological Rx
 Aspirin  <80yrs, stage 1 HTN (>140/90) and one of:
 Statin  Target organ damage (e.g. LVH, retinopathy)
 10yr CV risk ≥20%
Rx Precipitants / Causes  Established CVD
 Underlying cause  DM
 Valve disease  Renal disease
 Arrhythmias  Anyone with stage 2 HTN (>160/100)
 Ischaemia  Severe / malignant HTN (specialist referral)
 Exacerbating factors  Consider specialist opinion if <40yrs with stage 1 HTN
 Anaemia and no end organ damage.
 Infection
 ↑BP BP Targets
 Under 80yrs: <140/90 (<130/80 in DM)
 Over 80yrs: <150/90
Specific Mx
 ACEi, β-B and spiro → ↓ mortality CV Risk Mx
O
 Statins for 1 prevention if 10yr CVD risk ≥20%
1st line: ACEi/ARB + β-B + loop diuretic  Aspirin may be indicated: evaluate risk of bleeding
 ACEi/ARB: e.g. lisinorpil or candesartan
 Hydralazine + ISDN if not tolerated
Antihypertensive Rx
 β-B: e.g. carvedilol or bisoprolol (licensed)
 Start low, go slow < 55 > 55 / Black
 E.g. carvedilol 3.125mg/12h → 25-50mg/24h
 Wait ≥2wks between increments 1: A C (or D)
 Switch stable pts taking a β-B for a comorbidity to
a β-B licensed for heart failure
 β-B therapy may be particularly good in COPD 2: A + C (/D)

 Loop diuretic: frusemide or bumetanide


nd 3: A+C+D
2 line: get specialist advice
 Spironolactone / eplerenone
 Watch K carefully (on ACEi too…)
4: Resistant HTN
 ACEi + ARB  A+C+D+ consider further diuretic (e.g.
spiro) or α-blocker or β-B.
 Vasodilators: hydralazine + ISDN  Seek expert opinion
 Additional Rx in Blacks
rd
3 line A: ACEi or ARB
 Digoxin  e.g. lisinopril 10mg OD (↑ to 30-40mg)
 Cardiac resynchronisation therapy ± ICD  e.g. candesartan 8mg OD (max 32mg OD)
C: CCB: e.g. nifedipine MR 30-60mg OD
D: Thiazide-like diuretic: e.g. chlortalidone 25-50mg OD
Other Considerations
 Monitoring In step 2, use ARB over ACEi in blacks.
 BP: may be v. low Avoid thiazides + β-B if possible: ↑ risk of DM
 Renal function Only consider β-B if young and ACEi/ARB not tolerated.
 Plasma K
 Daily wt.
 Use amlodipine for comorbid HTN or angina Malignant HTN
 Avoid verapamil, diltiazem and nifedipine (short  BP > 180/110 + papilloedema and/or retinal
acting) haemorrhage
 Controlled ↓ in BP over days to avoid stroke
 Atenolol or long-acting CCB PO
 Encephalopathy / CCF: fruse + labetalol / nitroprusside IV
 Aim for 110 diastolic over ~4h
© Alasdair Scott, 2012 32
Respiratory
Contents
Bronchodilators............................................................................................................................................................................ 34 
Inhaled Corticosteroids ................................................................................................................................................................ 34 
Miscellaneous Respiratory Drugs................................................................................................................................................ 35 
Chronic Asthma ........................................................................................................................................................................... 36 
Chronic COPD ............................................................................................................................................................................. 36 

© Alasdair Scott, 2012 33


Bronchodilators

Drug MOA Side Effects Contraindications Interactions Other


β-agonists Act @ bronchial β2 receptors Tachycardia ↓ K in high doses c̄ Salbutamol may be
- SM relaxation Tremor - Corticosteroids given IVI in acute severe
Short-Acting, Fast Onset - ↓ mucus secretion - Loop / thiazide diuretics asthma.
- 2-4hrs - Theophylline
- Salbutamol: Ventolin
- Terbutaline: Bricanyl

Long-Acting
- 12-18hrs
- Salmeterol: Serevent
- Formoterol (Fast onset)
Muscarinic Antagonists Bronchodilatation Dry mouth Caution
↓ mucus secretion - Closed Angle Glaucoma
Short-Acting - Prostatic hypertrophy
- 3-6hrs
- Ipratropium: Atrovent

Long-Acting
- Tiotropium: Spiriva

Inhaled Corticosteroids

Drug MOA Side Effects Contraindications Interactions Other


Beclometasone: Becotide Act over wks → ↓ inflammation Oral candidiasis ↓ risk of complications
Budesonide: Pulmicort - ↓ cytokine production - use a spacer
Fluticasone: Flixotide - ↓ prostaglandin / leukotriene High doses may → typical - rinse mouth after use
synthesis steroid SEs
Symbicort - ↓IgE secretion Fluticasone is ~2x as potent
- Budesonide + formoterol - ↓ leukocyte recruitment as beclo or budes
- use lower dose
Seretide Prevent long-term ↓ in lung function
- Fluticasone + salmeterol Symbicort can be used as a
preventer or reliever
because of formoterol’s fast
onset.

© Alasdair Scott, 2012 34


Miscellaneous Respiratory Drugs

Drug MOA Side Effects Contraindications Interactions Other


Theophylline MR Methylxanthines Nausea ↓d levels Aminophylline is IV form
Aminophylline Arrhythmias - smoking - give IVI slowly
PDE inhibitors Seizures - EtOH - Too fast → VT
- ↑cAMP → bronchodilatation ↓K - CyP inducers - monitor c̄ ECG and
check plasma levels
↑d levels
- CCBs CyP metabolism
- CyP inhibitors
Montelukast: Singulair Leukotriene receptor antagonist ? Churg-Strauss Particularly useful for NSAID-
Zafirlukast - Block cysteinyl leukotrienes and exercise-induced asthma

Roflumilast PDE-4 inhibitor GI Sev immunological


disease
Omalizumab Humanised anti-IgE mAb SC injection every 2-4wks
Used for severe asthma
Carbocysteine Mucolytic GI bleed (rare) Active peptic ulceration Use: COPD
Dornase ALFA (DNase) Mucolytic Use: CF
Non-sedating Selective H1 R inverse agonists Hypotension CI: sev hepatic disease
- Certirizine - aka H1 antagonists Arrhythmia: ↑ QT
- Des-/ Loratidine: Clarityn Caution
- Fexofenadine Older agents - Long QT
- Drowsiness - BPH
Sedating - Anti-AChM - Closed-angle glaucoma
- Chlorphenamine: Piriton

© Alasdair Scott, 2012 35


Chronic Asthma Chronic COPD
General Measures: TAME Assess Severity
 Technique for inhaler use  Mild: FEV1 >80% (but FEV/FVC <0.7 and symptomatic)
 Avoidance: allergens, smoke (ing), dust  Mod: FEV1 50-79%
 Monitor: Peak flow diary (2x/d)  Severe: FEV1 30-49%
 Written instruction based on peak flow  Very Severe: FEV1 < 30%
 Educate
 Liaise c̄ specialist nurse General Measures
 Need for Rx compliance  Stop smoking
 Emergency action plan  Specialist nurse
 Nicotine replacement therapy
Drug Ladder  Bupropion, varenicline (partial nicotinic agonist)
 Support programme
1 SABA PRN  Pulmonary rehabilitation / exercise
 If use >1/d or nocte symptoms → step 2  Rx poor nutrition and obesity
 Screen and Mx comorbidities
2 Low-dose inhaled steroid: beclometasone 100-400ug bd  e.g. cardiovasc, lung Ca, osteoporosis
 200ug bd is good starting dose for most  Depression
 Influenza and pneumococcal vaccine
3 LABA: salmeterol 50ug bd  Review 1-2x/yr
 Good response: continue  Air travel risky if FEV1<50%
 Benefit but control still poor: ↑ steroid to 400ug bd
 No benefit: discontinue + ↑ steroid to 400ug bd Mucolytics
 Consider if chronic productive cough
If control is still poor consider trial of:  E.g. Carbocisteine (CI in PUD)
 Leukotriene receptor antagonist (e.g. monetelukast)
 Esp. if exercise- or NSAID-induced asthma Breathlessness and/or exercise limitation
 MR Theophylline  SABA and/or SAMA (ipratropium) PRN
 SABA PRN may continue at all stages
4 Trials of
 ↑ inhaled steroid to up to 1000ug bd Exacerbations or persistent breathlessness
 Leukotriene receptor antagonist  FEV1 ≥50%: LABA or LAMA (tiotropium) (stop SAMA)
 MR Theophylline  FEV1 <50%: LABA+ICS combo or LAMA
 MR β agonist PO
Persistent exacerbations or breathlessness
5 Oral steroids: e.g. prednisolone 5-10mg od  LABA+LAMA+ICS
 Use lowest dose necessary for symptom control  500ug fluticasone BD
 Maintain high-dose inhaled steroid  1000ug beclometasone BD
 Refer to asthma clinic  Roflumilast / theophylline (PDIs) may be considered
 Consider home nebs

LTOT
 Aim: PaO2 ≥8 for ≥15h / day (↑ survival by 50%)
 Clinically stable non-smokers c̄ PaO2 <7.3 (stable on two
occasions >3wks apart)
 PaO2 7.3 – 8 + PHT / cor pulmonale / polycythaemia /
nocturnal hypoxaemia
 Terminally ill pts.

Surgery
 Recurrent pneumothoraces
 Isolated bullous disease
 Lung volume reduction

© Alasdair Scott, 2012 36


Central Nervous System
Contents
Anti-Parkinsonian Drugs .............................................................................................................................................................. 38 
Anti-Epileptic Drugs ..................................................................................................................................................................... 39 
Anti-Migraine ............................................................................................................................................................................... 40 
Multiple Sclerosis Therapy .......................................................................................................................................................... 41 
Anti-Emetics ................................................................................................................................................................................ 42 
Anti-Depressants ......................................................................................................................................................................... 43 
Analgesia ..................................................................................................................................................................................... 44 
Other Psychiatric Drugs .............................................................................................................................................................. 45 
Migraine ....................................................................................................................................................................................... 46 
Epilepsy ....................................................................................................................................................................................... 46 
Parkinson’s Disease .................................................................................................................................................................... 47 
Drug-induced Movement Disorders............................................................................................................................................. 47 
Pain Control ................................................................................................................................................................................. 48 
Multiple Sclerosis......................................................................................................................................................................... 49 
Myasthenia Gravis ....................................................................................................................................................................... 49 
Psychiatry .................................................................................................................................................................................... 50 

© Alasdair Scott, 2012 37


Anti-Parkinsonian Drugs

Drug MOA Side Effects Contraindications Interactions Other


Levodopa Dopamine Pro-drug Dyskinesia Glaucoma (closed) fx ↓d by antipsychotics Always give c̄ peripheral dopa-
On-Off phenomena MAOIs decarboxylase inhibitor
Crosses BBB and converted to Da Psychosis Melanoma HTN crisis c̄ non-selective - Carbidopa (Co-careldopa)
by dopa-decarboxylase ABP↓ MAOIs - Benserezide (Co-beneldopa)
Mouth dryness Loss of response w/i 2-5yrs
Insomnia Anti-HTNs enhance ↓BP fx
N/V Give domperidone for n/v
EDS Food (protein) affects
absorption Short t½  at least TDS dosing
Apomorphine Non-selective Da agonist V. emetogenic Only give SC
- give domperidone for 2
days before starting Rx Rescue pen for “off” freezing
Injection site reactions
Bromocriptine Ergot-derived Da agonists Fibrosis Cardiovascular disease Levels ↑d by: Not often used in parkinsonism
Cabergoline Vasospasm: cardiac, digital Porphyria - octreotide due to SEs
Pergolide GI upset Psychosis - macrolides
Postural hypotension
Drowsiness
Neuropsych syndromes
Ropinirole Synthetic Da agonists GI upset
Rotigotine Drowsiness
Pramipexole Postural hypotension
Neuropsych syndromes
Selegiline Selective MAO-B inhibitors GI upset Used alone to delay need for L-
Rasagiline Prevent intraneuronal degradation Insomnia (selegiline) dopa
of Da Postural hypotension Adjunct to L-dopa to ↓ end-of-
Buccal preps → better bioavailability (no cheese reaction) dose effects
Entacapone COMT inhibitors Reddish-brown urine Interact c̄ ↓ “off” period of L-DOPA
Tolcapone GI disturbance sympathomimetics
Inhibit peripheral Da degradation Dyskinesias Tolcapone has better efficacy
Tolcapone → hepatotoxic but requires LFT monitoring.
Amantadine → Da release GI upset Gastric ulcer May be used in PD for late-
Weak anti-cholinergic Sleep disturbance Epilepsy onset dyskinesia
Livedo reticularis
Neuropsych syndromes
Procyclidine Muscarinic antagonists Anti-AChM Useful in drug-induced
Benzhexol Memory impairment parkinsonism and mild PD in
Reduce tremor Confusion young pts: esp. tremor

L-DOPA Motor Fluctuations


 Peak dose dyskinesias
 End-of-dose dyskinesia / akinesia: deterioration as dose wears off c̄ progressively shorter benefit. 
 On-Off effect: unpredictable fluctuations in motor performance unrelated to timing of dose.
© Alasdair Scott, 2012 38
Anti-Epileptic Drugs

Drug MOA Side Effects Contraindications Interactions Other


Valproate Na+ channel blockers GI upset Acute porphyria fx ↓d by:
st O
1 line for 1 generalised
Use-dependent Hepatotoxicity - antimalarials seizures
Inhibit action potential generation Personal/fam hx of - antidepressants
Appetite ↑ → ↑wt. severe liver dysfunction - antipsychotics CYP inhibitor
Liver failure - some anti-epileptics
Pancreatitis L/P Monitor
Reversible hair loss levels ↑d by cimetidine - FBC, LFTs
Oedema
Ataxia ↑s fx of: Most teratogenic AED
Teratogenicity, Tremor, - aspirin - 6% overall
Thrombocytopenia - lamotrigine
Encephalopathy: due to ↑ - warfarin
ammonia

Carbamazepine Skin reactions (e.g. SJS) Unpaced AV conduction ↓s fx of: Has active metabolite produced
- Tegretol Blood dyscrasias (↓WCC) defects - COCP in the liver
↓Na+ (SIADH) - Doxy
Foetal NTDs Hx of BM depression - corticosteroids CYP inducer
GI upset - anti-epileptics (inc. CBZ)
Porphyria - nifedipine Monitor
Dose-related - Warfarin - se levels
- Dizziness/vertigo MAOIs - U+Es, LFTs, FBC
- Ataxia levels ↑d by:
- Diplopia - macrolides
Phenytoin Acute Don’t give IV if cardiac - cimetidine V. albumin bound
- Drowsiness dysrhythmias - diltiazem and verapamil
- Cerebellar fx (DANISH) - EtOH CYP inducer
- Rash Caution: DM, ↓BP, L/H, - NSAIDs
P (cleft palate) - esomeprazole Saturable kinetics
Chronic - dose ↑ → t½ ↑
- Gingival hyperplasia levels ↓d by:
- Hirsutism + acne - rifampicin Monitor
- ↓ folate - antipsychotics - FBC
Lamotrigine Inhibits glutamate release Rashes (SJS, TEN, lupus) L fx ↓d by: Monitor
Cerebellar fx - OCP - U+Es, LFTs, FBS, clotting
Blood dyscrasias - phenytoin, CBZ
Hepatotoxic - TCAs and SSRIs Stop if any sign of a rash

levels ↑d by valproate Safest drug for pregnancy


Ethosuximide Ca2+ channel blocker GI upset ?makes tonic-clonics Only used for childhood
worse absence seizures
Vigabatrin Visual field defects

© Alasdair Scott, 2012 39


Anti-Migraine

Drug MOA Side Effects Contraindications Interactions Other


Rizatriptan 5HT1B/1D receptor agonist Sensations of tingling/heat/ IHD ↑ risk of CNS toxicity c̄ Used for Rx of acute attacks
Sumatriptan tightness/pressure Coronary vasospasm SSRIs
Reverses dilatation of cerebral PVD Don’t use >2-3x / wk
vessels Dizziness Hx of MI, CVA, TIA - → chronic migraine
Flushing HTN (mod/sev)
Ergotamine Partial 5HT1 receptor agonist GI upset IHD Use limited by SEs
Dizziness Coronary vasospasm
PVD
HTN (mod/sev)
Hx of MI, CVA, TIA
Pizotifen 5HT2 receptor antagonist and Drowsiness Prophylaxis
antihistamine ↑ appetite and ↑ wt.
Amitriptylline 5HT and NA reuptake inhibitor Anti-cholinergic Recent MI (w/i 3mo) MAOIs → HTN and CNS Hepatic metabolism
Anti-adrenergic Heart block excitation
Anti-histamine L Prophylaxis
Levels ↑d by
OD: prolonged QT → TDP - SSRIs
- Cimetidine

↑ risk of arrhythmias c̄
amiodarone.

© Alasdair Scott, 2012 40


Multiple Sclerosis Therapy
Drug MOA Side Effects Contraindications Interactions Other
Methylpred Inhibits PLA2 → ↓PG and ↓ PAF Cushing’s High dose (up to 1g/day) for
↓PMN extravasation → ↑PMN in blood - DM acute flares.
Lymphopaenia - Central obesity
↓Phagocytosis - Dyslipidaemia Short course (3-5d ays)
↓ Ab production - PUD
↓ cytokine and proteolytic enzyme - Osteoporosis
- Hirsutism, acne
Interferon-β1 Flu-like symptoms Decompensated L Relapsing remitting or
Injection site reaction Severe depression secondary progressive MS

Monitor for hepatotoxicity


Glatiramer Random polymer of amino acids found Flu-like symptoms Relapsing remitting MS
in myelin basic protein Injection site reaction
? acts as decoy
Natalizumab anti-α4 integrin
Alemtuzumab anti-CD52
Baclofen GABAB agonist Sedation PUD fx ↑d by TCAs Rx painful muscle spasms
↓ tone
Skeletal muscle relaxant Nausea Don’t withdraw abruptly
Urinary disturbance - hyperthermia
- ↑ spasticity
Dantrolene Prevents Ca2+ release from Hepatotoxicity L
sarcoplasmic reticulum GI upset

Skeletal muscle relaxant


Oxybutynin Antimuscarinic Dry mouth Myasthenia Used for detrusor instability
GI upset GI/bladder obstruction
Blurred vision

© Alasdair Scott, 2012 41


Anti-Emetics
Drug MOA Side Effects Contraindications Interactions Other
Metoclopramide D2-receptor antagonist EPSEs <20yrs ↑ risk of EPSEs c̄ Indications:
Prochlorperazine Prokinetic action in GIT - Dystonias GI obstruction antipsychotics, TCAs and - GI causes – esp. GORD
Domperidone - → ↑ absorption of other drugs - Oculogyric crisis L SSRIs. - Chemo, morning-after-pill, opiates
- PD
Drowsiness Prolactinoma - Migraine
Rash, allergy - Vestibular (prochlorperazine)
↑ prolactin
Domperidone doesn’t cross BBB
 less EPSEs cf. others
Ondansetron 5HT3-receptor antagonist Constipation Avoid if ↑QTc Levels ↓d by: Indications:
Granisetron Headache - Rifampicin - Post-op
- CBZ, phenytoin - Chemo

Avoid c̄ drugs that ↑ QTc CYP metabolism


Cyclizine H1-receptor antagonist Anti-AChM Severe HF MOAIs can → ↑↑ Indications:
Cinnarizine MOAIs antimuscarinic fx - Opioids (but not ACS)
- Vestibular
Hyoscine Anti-muscarinic Anti-muscarinic Glaucoma (closed- ↓s fx of SL GTN Indications:
hydrobromide angle) - Prophylaxis vs. motion sickness
BPH - Hypersalivation
Dexamethasone Steroid – unknown anti-emetic effect Indications:
- Chemo (adjunct)
- Surgery
Aprepitant Neurokinin receptor blocker Indications:
- Chemo (adjunct)

Causes of N/V Physiology


 Drugs Vomiting regulated by Vomiting Centre and CTZ, both located in the medulla.
 Abx: e.g. erythromycin
 Anti-parkinsonian: e.g. L-DOPA CTZ
 Cytotoxic agents  Outside BBB  accessible to drugs Emetogenic Receptors:
 Opioids  Also receives input from vestibular system re. motion  H1
 Digoxin  Expresses: D2 and 5-HT3 receptors  D2
 GI: GORD, gastro, pancreatitis, obstruction  CTZ projects to Vomiting Centre  5HT3
 Neoplasia: oesophageal, gastric, duodenal  mACh
 Vestibular: e.g. labyrinthitis, sea-sickness Vomiting Centre
 Neuro: e.g. migraine, ↑ICP  Controls visceral and somatic functions involved in vomiting.
 Vagal: e.g. pancreatitis, MI, torsion  Receives input from CTz
 Pregnancy  Also receives muscarinic and histaminergic input (H1)

© Alasdair Scott, 2012 42


Anti-Depressants

Drug MOA Side Effects Contraindications Interactions Other


Paroxetine SSRI N / V / diarrhoea Active mania P450 inhibitor → ↑s levels of Takes 4-6wks for full clinical effect
Citalopram Insomnia Children <18yrs - TCAs Don’t stop suddenly
Fluoxetine Headache - except fluoxetine - benzos Avoid w/i 2wks of MAOI
Sertraline Sexual dysfunction - clozapine, haldol
SIADH - CBZ and phenytoin Use
Withdrawal effects - Depression
SSRI + MAOI → serotonin syndrome - OCD
- Eating disorders
↑ risk of bleeding c̄ aspirin - Anxiety disorders
Venlafaxine SNRI HTN HF (3/4) SSRI + MAOI → serotonin syndrome 2nd line anti-depressant
GI upset Uncontrolled HTN
↑QTc ↑ risk of bleeding c̄ aspirin Stop drug if signs of rash
SIADH
Rash
Amitriptyline TCA α1 Recent MI MAOIs → HTN and CNS excitation Avoid w/i 2wks of MAOI
Lofepramine - postural hypotension Arrhythmias
Clomipramine Inhibit 5HT and NA uptake - sedation Severe L Levels ↑d by SSRIs
Imipramine Mania
Doxepin H1 ↑ risk of arrhythmias c̄ amiodarone
Norttriptyline - drowsiness Caution
- wt. gain - Glaucoma TCAs lower the seizure threshold
- BPH - ↓ effects of AEDs
Anti-AChM
↑ fx of antipsychotics
Arrhythmias
- esp. heart block
Phenelzine MAOI Sedation Phaeo Hypertensive Crisis Moclobemide is reversible and is
Isocarboxacid Hypotension - tyramine containing foods selective for MAOI-A  less chance
Moclobemide (A) Inhibit monoamine Anti-AChM - opioids: esp. pethidine of interactions.
Selegiline (B) metabolism
- A: 5HT SSRIs and TCAs → serotonin syn.
- B: Da

Serotonin Syndrome TCA Toxicity


 Cognitive: headache, agitation, confusion, coma  Metabolic acidosis
 Autonomic: sweating, ↑HR, palpitations, HTN, hyperthermia  Anti-AChM: dilated pupils
 Somatic: myoclonus, clonus, hypertonia, tremor  CNS: hypertonia, hyperreflexia, extensor plantars, seizures
 Cardiac: ↑HR, ↑QTc → TDP
 Pulmonary: hypoventilation
Monoamine Oxidase  Rx: NaHCO3
 Metabolises monoamines
 MAO-A: adrenaline, norad, serotonin, tyramine, dopamine
 MAO-B: dopamine
© Alasdair Scott, 2012 43
Analgesia

Drug MOA Side Effects Contraindications Interactions Other


Paracetamol Antipyretic Hepatic failure in OD Severe L
Analgesic
Strong opioids Analgesic effect mediated CNS Avoid in pts. c̄ acute Rx overdose c̄ naloxone
- Morphine by u receptor. - Resp. depression respiratory depression
- Diamorphine - Sedation ↓ dose in
- Buprenorphine - Nausea / vomiting Head injury - renal impairment
- Fentanyl - Euphoria - can’t assess pupils - hepatic impairment
- Pethidine - Meiosis - elderly
- Oxycodone - Anti-tussive
- Dependence
Weak opioid
- Codeine Non-CNS
- Dihydrocodeine - Constipation
- Tramadol - Urinary retention
- Pruritis
- Bradycardia, hypotension
Gabapentin Unknown Sedation fx ↓d by: Pregabalin is more potent
Cerebellar fx - antidepressants gabapentin analogue
Dizziness - antimalarials
Peripheral oedema
Amitriptyline TCAs
Nortroptyline

© Alasdair Scott, 2012 44


Other Psychiatric Drugs

Drug MOA Side Effects Contraindications Interactions Other


Li Mood stabiliser Polyuria and polydipsia Hypothyroidism Toxicity ↑d by Monitor
Nephrotoxic P/R/H - NSAIDs - drug levels
GI upset - Diuretics (esp. thiazides) - U+Es
Fine tremor - ACEi / ARB - TFTs
Hypothyroidism
Use
Toxicity - Acute mania
- Coarse tremor - Prophylaxis of BAD
- Cerebellar signs - Resistant depression
- AKI
- Hyper-reflexia ↑ toxicity when ↓ Na or dehydrated
- Coma - ↑ Li reabsorption in renal PCT
Chlorpomazine Typical antipsychotics Sedation fx ↑d by: Monitor FBC, U+E, LFTs
Haldol Da antagonists Anti-AChM - Li
Sulpiride EPSEs - TCAs
Zuclopenthixol Neuroleptic malignant syn.
↑QTc, postural hypotension
↑ PRL
Sexual dysfunction
↑ wt.
Clozapine Atypical antipsychotics Clozapine: agranulocytosis, ↑ wt, DM Can still → EPSEs @ high doses
Olanzapine Da antagonists - apart from clozapine
Quetiapine Olanzapine: ↑ wt., DM, sedation
Risperidone Clozapine
Quetiapine: sedation - Rx of refractory schizophrenia
- better @ Rx of negative symp
Risperidone: ↑wt,, ↑ PRL
Benzodiazepines Promote GABA Sedation Resp depression levels / fx ↑d by: Rx overdose with flumazenil
binding to GABAA Respiratory depression - antipsychotics
receptors Withdrawal - azoles Hepatic metabolism
- macrolides
IV diazepam is given as an emulsion
to ↓ risk of thrombophlebitis
Phenobarbitol Potentiate GABAA Sedation See CBZ, phenytoin. CYP inducer
receptors Respiratory depression Primidone is phenobarbital prodrug

© Alasdair Scott, 2012 45


Migraine Epilepsy
Acute episode Seizure Type 1st line 2nd line
 1st: Paracetamol + metoclopramide / domperidone Generalised Valproate Levitiracetam
 2nd: NSAID (e.g. ketoprofen) + M/D - tonic-clonic Lamotrigine
 3rd: Rizatriptan (5-HT1B/1D agonist) - absence
 CI: IHD, uncontrolled HTN, SSRIs - myoclonic
 4th: ergotamine Focal onset Lamotrigine Levitiracetam
Carbamazepine

Prophylaxis
 Avoid triggers In Women / Pregnancy
 1st: Propanolol, amitriptyline, topiramate  Avoid valproate: take lamotrigine (or CBZ)
 2nd: Valproate, pizotifen (↑ wt.), gabapentin  5mg folic acid daily if child-bearing age
 CBZ and Phenytoin are enzyme inducers
 ↓ effectiveness of the OCP
 Oral vit K in last month

Driving Advice
 Must not drive w/i 12mo of seizures
 Pts. who only have seizures while sleeping for ≥3yrs
can drive.
 Person must comply c̄ Rx

© Alasdair Scott, 2012 46


Parkinson’s Disease Drug-induced Movement Disorders
General Causes
 MDT: neurologist, PD nurse, physio, OT, social  Neuroleptics
worker, GP and carers  Anti-emetics: metoclopramide, prochloperazine
 Assess disability  L-DOPA → acute dystonias
 e.g. UPDRS: Unified Parkinson’s Disease Rating Scale
 Physiotherapy: postural exercises Mx
 Depression screening
Disorder Rx
Acute Dystonia Procyclidine
Medical Parkinsonism Procyclidine
Da agonists
Young onset ± biologically fit  Akathisia Propranolol
1. Da agonists: ropinirole, pramipexole  Tardive dyskinesia Switch to atypical anti-
2. MOA-B inhibitors: rasagiline, selegiline  psychotic
3. L-DOPA: co-careldopa or co-beneldopa Neuroleptic malignant syn. Dantrolene
Da agonists: e.g apomorphine
Biologically frail ± comorbidities 
1. L-DOPA 
2. MOA-B inhibitors  Other Movement Disorders
 Restless legs: ropinerole
Response Fluctuations  Benign essential tremor: propranolol
 MOA-B inhibitors  Focal dystonias: botulinum toxin, benzos
 ↓ end-of-dose effects  Blepharospasm
 COMT inhibitor: tolcapone, entacapone  Spasmodic torticolis
 Lessen “off” time @ end-of-dose  Writer’s cramp
 Apomorphine: potent Da agonist
 SC rescue pen for sudden “off” freezing
 Amantidine: weak Da agonist
 Rx of drug-induced dyskinesias

Adjunctive Therapies
 Domperidone
 Rx of drug-induced nausea
 Atypical antipsychotics: e.g. quetiapine, clozapine
 Disease-induced psychosis
 SSRIs: citalopram, sertraline
 Depression

Surgical
 Interrupt basal ganglia
 Deep brain stimulation
 Stem cell Transplant

© Alasdair Scott, 2012 47


Pain Control
Principles Neuropathic Pain
 Oral where possible
 Fixed interval to give continuous relief Common Causes
 Stepwise approach  DM
 EtOH
 Ca
WHO Analgesia Ladder  Trigeminal neuralgia
 Post-herpetic neuralgia
1. Non-opioid ± adjuvants  HIV
 Paracetamol
 NSAIDs
 Ibuprofen: 400mg/8h PO Main Options
st
 Diclofenac: 50mg PO / 75mg IM  1 : pregabalin / gabapentin
 2nd: TCAs
2. Weak opioid + non-opioid ± adjuvants  3rd: opioids
 Codeine  4th: CBZ, valproate, lamotrigine
 Dihydrocodeine
 Tramadol
Topical: lidocaine patches, capsaicin
3. Strong opioid + non-opioid ± adjuvants
 Morphine: 5-10mg/2h max NB. HIV-assoc. sensory neuropathy responds better to
 Oxycodone gabapentin than amitriptyline.
 Fentanyl

NSAIDs
 → gastric and duodenal ulceration
 Na and H2O retention
 Worsen heart failure
 Interfere c̄ ACEi / ARB
 May worsen / precipitate asthma
 COX-2 selectives ↑ CV risk

Potent Opioids
 Establish dose c̄ intermediate release preparations then
give maintenance c̄ modified release
 Start c̄ oramorph 4-10mg/4h PO c̄ =
breakthrough dose PRN
 Switch to modified release (MST) BD: BD dose =
total 24h dose / 2.
 Give 1/6 total daily dose as oramorph for
breakthrough pain
 Consider PCA

Mx of SEs
 Constipation: codanthrusate (stimulant laxative)
 Nausea: metoclopramide
 Drowsiness: tolerance develops

Other Options
 Nerve blocks: visceral pain
 Direct local anaesthetic injections: facet joint pain

Adjuvants
 Anything not specifically mentioned on the ladder
 Surgery
 Chemo

© Alasdair Scott, 2012 48


Multiple Sclerosis Myasthenia Gravis
MDT: neurologist, radiologist, physio, OT, specialist nurses,
GP, family Diagnosis: Tensilon Test
 Edrophonium bolus IV
 Positive = improvement of power w/i 1min
Acute Attack
 Methylpred 1g IV/PO /24h for 3d
 Doesn’t influence long-term outcome
Mx
 ↓ duration and severity of attacks
Symptom Control
 Anticholinesterase: e.g. pyridostigmine.
 Cholinergic SEs
Preventing Relapse
Immunosuppression
DMARDs
 Rx relapses c̄ pred
 IFN-β: ↓ relapses by 30% in relapsing remitting MS
 Steroids ± sparing-agents
 Glatiramer: similar efficacy to IFN-β
 Azathioprine, methotrexate, cyclosporin
 Can → total remission
Biologicals
 Natalizumab: anti-VLA-4 Ab
Thymectomy
 ↓ Relapses by 2/3 in RRMS
 Consider if young onset and disease not control by
 Alemtuzumab: anti-CD52
nd anticholinesterases
 2 line in RRMS
 Always remove a thymoma if present
 May be malignant
 Remission in 25%, benefit in further 50%.
Symptomatic
 Fatigue: modafinil Myasthenic Crisis
 Depression: SSRI (citalopram)  Plasmapheresis
 Pain: amitryptylline, gabapentin  IVIg
 Spasticity: physio, baclofen, dantrolene, botulinum
 Urgency / frequency: oxybutynin, tolterodine
 ED: sildenafil
 Tremor: clonazepam

© Alasdair Scott, 2012 49


Psychiatry
Antidepressants
SSRIs: selective serotonin reuptake inhibitors
 Citalopram, sertraline, fluoxetine

SNRIs: serotonin noradrenaline reuptake inhibitors


 Venlafaxine

NaSSA: Norad and specific serotonergic antidepressant


 Mirtazapine

NRI: Norad reuptake inhibitor


 Reboxetine

TCAs: tricyclic antidepressants


 Amitriptyline
 Imipramine
 Clomipramine

MAOIs: monoamine oxidase inhibitors


 Meclobemide (RIMA)
 Phenelzine

Bipolar Affective Disorder


 Lithium (TDM essential)
 AEDs: valproate, CBZ
 Neuroleptics: olanzapine

Antipsychotics
Typical
 Haldol
 Chlopromazine

Atypical
 Clozapine
 Olanzapine
 Risperidone
 Quetiapine

© Alasdair Scott, 2012 50


Infection
Contents
Beta Lactams............................................................................................................................................................................... 52 
Inhibitors of Protein Synthesis ..................................................................................................................................................... 53 
Other Antibiotics .......................................................................................................................................................................... 54 
Anti-Malarials ............................................................................................................................................................................... 55 
Anti-Virals .................................................................................................................................................................................... 55 
Anti-Retrovirals ............................................................................................................................................................................ 56 
Anti-Fungals ................................................................................................................................................................................ 57 
Bacterial Infections ...................................................................................................................................................................... 58 
Viral Infections ............................................................................................................................................................................. 59 
Protozoal Infections ..................................................................................................................................................................... 60 
Fungal Infections ......................................................................................................................................................................... 60 

© Alasdair Scott, 2012 51


Beta Lactams

Drug MOA Use Side Effects CIs Interactions


Penicllins Bactericidal Pen V Streps Hypersensitivity Hypersensitivity - May ↓ fx of OCP
- Inhibit bacterial transpeptidase enzyme Pen G (IV) N. meningitidis - Rash 10% x-reactivity
Required for cell wall construction Syphilis - EM c̄ cephs ↑d by probenecid
Broad Spectrum Pneumococcus - anaphylaxis
- Amoxicillin Listeria
- Ampicillin E. coli GI upset
Enterococci
Penicillinase-resistant MSSA Mac pap rash c̄ EBV
- Fluclox
Anti-pseudomonal Pseudomonas
- Piperaccillin
- Ticarcillin
Co-amoxiclav Severe CAP
UTI
Tazocin Severe HAP
Sepsis
Cephalosporins Bactericidal 1st gen UTI GI upset Hypersensitivity
- Inhibit bacterial transpeptidase enzyme - Cephalexin AAC
Required for cell wall construction - Cefaclor
2nd gen Mod / sev CAP
Generations have ↑ing activity vs. Gm-ve - Cefuroxime GI sepsis
Pre-op
3rd gen Meningitis
- Cefotaxime Epiglottitis
- Ceftriaxone Gonorrhoea
- Ceftazidime SBP
- Cefixime
Carbapenems Bactericidal Imipenam All Gm+ except MRSA GI upset
- Inhibit bacterial transpeptidase enzyme Meropenam Most Gm-ves
Required for cell wall construction Ertapenam Neutropenic sepsis

V. broad spectrum
- Gm-, Gm+ and anaerobes
- Pseudomonas

Imipenem is rapidly inactivated by the


kidney and must be given c̄ cilastatin
which blocks its metabolism.

© Alasdair Scott, 2012 52


Inhibitors of Protein Synthesis

Drug MOA Use Side Effects CIs Interactions Other


Chloramphenicol Bacteriostatic Conjunctivitis Irreversible aplastic PLR
- 50s subunit anaemia
Grey baby syndrome
Gentamicin Aminoglycosides Gm- sepsis Nephrotoxic Myasthenia gravis ↓ absorption c̄ Must monitor levels
Amikacin Neutropenic sepsis Ototoxic - milk - peak and trough
Tobramycin Bactericidal Otitis externa Caution in R - antacids
Neomycin Amino-acyl site of 30s - alter dose and time Must be given IV
Streptomycin subunit Anti-pseudomonal
Tetracycline Tetracyclines COPD exacerbation GI upset Children <12yrs Toxicity ↑ by
Doxycycline Acne Hypersenstivity L - frusemide
Bacteriostatic Chlamydia Bone deposition R - cephs
30s subunit Rickettsia - vanc
Brucella - ciclosporin
Lyme disease
Linezolid Oxazolidinones MRSA and VRE Blood dyscrasias Caution in R and L Linezolid is a non-
selective MAOI
Bacteristatic No activity vs. Gm- - avoid SSRI, TCAs
23s component of 50s and Tyramine
subunit
Monitor FBC
Erythromycin Macrolides Pen allergy ↑ QTc Caution if ↑QTc P450 inhibitor Also have GI
Clarithromycin Atypical pneumonia Dry skin - W+ prokinetic action
Azithromycin Bacteriostatic Chlamydia Cholestatic hepatitis
50s subunit H. pylori ↑ digoxin
Synercid Streptogramins VRE Only used when other
MRSA agents failed.
Bacteriostatic
50s subunit

Clindamycin Lincosamides Active vs. Gm+ cocci and AAC Diarrhoea Stop drug if pt.
bacteroides Hepatotoxicity develops diarrhoea
Bacteriostatic
50s subunit Osteomyelitis
MRSA

© Alasdair Scott, 2012 53


Other Antibiotics

Drug MOA Use Side Effects CIs Interactions Other


Vancomycin Glycopeptides Aerobic and anaerobic Gm+ Nephrotoxic ↓ dose in renal Must monitor levels
Teicoplanin Bactericidal MRSA Ototoxic impairment - pre-dose trough level
Inhibits cell wall synthesis HAN - tinnitus
Unable to penetrate Gm- Infective endocarditis - SNHL
outer cell wall AAC (PO) Hypersensitivity rash
Poor oral absorption Neutropenia
Ciprofloxacin Fluoroquinilones Broad spectrum: esp. Gm- ↑ QTc P P450 inhibitor
Levofloxacin Bactericidal GI infections: campy, shig… GI upset
Ofloxacin Inhibit DNA synthesis Pseudomonas: esp. in CF Tendonitis ± rupture Antacids → ↓
Moxifloxacin Prostatitis, PID ↓ seizure threshold absorption
Anthrax Photosensitivity
Metronidazole Nitroimidazole Anaerobes Metallic taste Avoid EtOH Aldehyde dehydrogenase
Nitrofurantoin Bactericidal GI sepsis GI upset - Disulfiram-like inhibitor
Tinidazole Inhibits DNA synthesis Aspiration pneumonia Metro: gynaecomastia reaction
AAC
H. pylori
PID
Protozoa:Giardia
Rifampicin Rifamycins Mycobacteria Yellow secretions Jaundice P450 inducer Rifaximin has v. poor oral
Rifaximin Bactercidal Legionella Hepatitis - W- absorption and is  used in
Rifabutin Inhibit RNA synthesis Prophylaxis vs. meningits - ↓OCP hepatic encephalopathy.
- ↓AEDs
Trimethoprim Bacteriostatic UTI Blood dyscrasias Severe R and L Stop immediately if rash or
Sulfamethoxazole Folate antagonists PCP EM → SJS P dyscrasias occurs.
Sulfadiazine Toxoplasmosis EN
Co-trimoxazole Nephro- + hepato-toxicity
Daptomycin Cell membrane toxin MRSA
- Alternative to linezolid and
syndercid
Colistin Cell membrane toxin Active vs. Gm- Myasthenia gravis
Inhaled for CF
Ethambutol Bacteriostatic Anti-TB Optic neuritis Monitor vision
Inhibits mycobacterial cell - colour vision goes first
wall synthesis
Pyrazinamide Bactericidal Anti-TB Hepatitis Caution in gout Monitor LFTs
Gout
Isoniazid Bacteriostatic Anti-TB Peripheral neuropathy P450 inhibitor ↑ risk of SEs if slow
Hepatitis acetylator

Give c̄ pyridoxine
Fusidate Bacteriostatic Active vs. staphs Hepatitis Needs 2nd Abx to prevent
Impetigo (topical) resistance
Blepharitis (topical)
© Alasdair Scott, 2012
Osteomyelitis (PO) 54
Anti-Malarials
Drug Use Side Effects CIs Interactions Other
Chloroquine Benign malaria Visual change: rarely retinopathy Caution in G6PD deficiency
Prophylaxis Seizures
EM → SJS
Primaquine Benign malaria Haemolysis if G6PD deficient Caution in G6PD deficiency
- eliminate liver stage Methaemoglobinaemia
Malarone Falciparum malaria Abdo pain Avoid in renal impairment if
- proguanil + atovaquoone Prophylaxis Gi upset possible
Mefloquine Prophylaxis Nausea, dizziness Hx of epilepsy or psychosis Avoid if low risk of resistance
Neuropsychiatric signs
Riamet Falciparum malaria ↑QTc Hx of arrhythmias
- artemether + lumefantrine Abdo pain ↑QTc
GI upset Caution in R L

Anti-Virals

Drug MOA Use Side Effects CIs Interactions


Aciclovir Guanosine analogue Genital herpes GI upset Caution in renal
Phosphorylated by viral thymidine kinase Herpes meningitis ARF impairment
Di- and tri-phosphorylated by cellular kinase Herpes zoster Encephalopathy
Aciclovir triphosphate inhibits viral DNA pol Varicella zoster
Aciclovir is poor substrate for host DNA pol and TK Bell’s palsy
Valaciclovir Aciclovir prodrug
Converted → aciclovir by hepatic esterases during 1st pass
metabolism
Better oral bioavailability
Famciclovir Pro-drug c̄ same MOA as aciclovir
Ganciclovir 2-deoxyguanosine analogue CMV Rx BM suppression ↑↑ risk of BM suppression
Phosphorylated to dGTD analogue by viral UL97 - retinitis c̄ zidovudine
Triphosphate competitively inhibits viral DNA pol - pneumonitis
IV only CMV prophylaxis
Valganciclovir Ganciclovir prodrug with better oral bioavailabilty HHV-6 Tx disease
Foscarnet Binds to pyrophosphate-binding site and inhibits viral DNA pol CMV Rx Nephrotoxic Renal Tx
Doesn’t require viral TK - avoid in renal Tx
IV only
Cidofovir Inhibits viral DNA polymerase Resistant CMV Nephrotoxic
No activation required infections

© Alasdair Scott, 2012 55


Anti-Retrovirals

Class Antiviral MOA Side Effects Interactions


NRTI Emtricitabine NucleoSide Reverse Transcriptase Inhibitors Hepatitis
Stavudine - stop if ↑ LFTs
Tenofovir Except Tenofovir which is nucleoTide reverse transcriptase inhibitor Lactic Acidosis (type B)
Abacavir Painful peripheral neuropathy
Didanosine Rash
Lamivudine GI disturbance
Zidovudine
PIs Ritonavir Inhibit viral protease required for virus assembly Metabolic syndrome P450 inhibitors
Ritonavir is used to boost levels of other PIs Lipodystrophy
Indinavir

Saquinavir

Lopinavir / ritonavir
“Kaletra”
NNRTI Efavirenz Non-competitive inhibition of reverse transcriptase Insomnia, vivid dreams,
Nevirapine NB. Nevirapine is used to prevent HIV transmission during pregnancy dizziness
EM → SJS
Integrase Raltegravir Inhibit integration of transcribed viral DNA into host genome
inhibitors Elvitegravir
CCR5 inhibitor Maraviroc Binds CCR5 preventing interaction with gp120
Inhibits attachment of HIV
Fusion inhibitor Enfuviritide Binds gp41 and inhibits fusion Hypersensitivity at injection site

Indications Lipodystrophy
 CD4 ≤350  Fat redistribution
 AIDS-defining illness  ↓ SC fat
 Pregnancy  ↑ abdo fat
 HIVAN  Buffalo hump
 Co-infected c̄ HBV when Rx is indicated for HBV  Insulin resistance
 Dyslipidaemia
Use 2 NRTIs + 1 NNRTI or PI

Immune Reconstitution Inflammatory Syndrome


 Improvement in immune function 2O to ARV Rx
 Marked inflammatory reaction vs. residual opportunistic organisms
 Paradoxical worsening of symptoms on initiation of ARVs

© Alasdair Scott, 2012 56


Anti-Fungals

Class Drug Mechanism Indication SEs Misc


Polyenes Amphotericin B Interacts with ergosterol  pore formation Severe systemic fungal infections (IV) Nephrotoxic (IV) Monitor Cr
- cryptococcal meningitis
Fungicidal - pulm. Aspergillosis IV reaction (after 1-3h) PO version is
- systemic candidiasis - fever non-toxic
- hypotension
- nausea/vomiting
Nystatin Candidiasis: cutaneous, vaginal, Toxic if given IV PO or topical
mucosal, oesophageal
Imidazoles Ketoconazole Blocks ergosterol synth by inhibiting 14α- Chronic mucocutaneaous candidiasis Hepatotoxic
demethylase  ↓ membrane fluidity ↓androgen synthesis
Miconazole Dermatophyte infections
Clotrimazole Inhibits replication Mucocutaneous candidiasis
Triazoles Fluconazole Oral/vag/oesophagus candida P450 inhibitor
Prevents hyphae formation Alternative to ampho B for systemic
infections
Itraconazole Broad spectrum Blasto/histo/coccidio
Sporotrichosis
Fungistatic Chromomycosis
Aspergillus
Voriconazole Invasive candida or aspergillus in Photophobia
immunocomp Rash
BMT transplant pt. prophylaxis Hepatotoxic
Posaconazole Invasive candida, mucor and
aspergillus in immunocomps
Allylamines Terbinafine Blocks ergosterol synth by inhibiting Dermatophyte Infections GI effects
squalene epoxidase  membrane disruption Hives
↑LFTs
Fungicidal Reversible agranulocytosis
Echinocandins Caspofungin Inhibit β-glucan synthesis Invasive aspergillosis or candidiasis V. low toxicity IV only

Fungicidal vs. yeasts Empiric Rx for fungal infection in febrile GI upset


neutropenia Hypersensitivity
Fungistatic vs. moulds
Flucytosine Inhibits DNA/RNA synthesis Crptococcal meningitis (in combo with Bone marrow suppression
amphotericin B) Deranged LFTs
Griseofulvin Disrupts spindle formation in mitosis Dermatophyte infections of Very slow acting
skin/hair/nails

© Alasdair Scott, 2012 57


Bacterial Infections
Pneumonias Meningitis
 Community: benpen 1.2g IV/IM
CAP  <50: ceftriaxone 2g IVI/IM BD
 >50: ceftriaxone + ampicillin 2g IVI /4h
Mild amoxicillin 500mg TDS PO for 7d or  Viral suspected: aciclovir
clarithro 500mg BD PO for 7d  Give Dex 0.15mg/kg/6h c̄ first dose of Abx unless in
septic shock or meningococcal sepsis
Mod amoxicillin 500mg TDS and clarithro 500mg BD PO/IV
for 7d (clarithro alone if pen allergy)
Urinary Tract Infection
Sev Co-amoxiclav 1.2g TDS IV / cefuroxime 1.5g TDS IV  Pyelonephritis: cefotaxime 1g IV BD for 10d
and clarithro 500mg BD IV for 7-10d  Lower UTI: Rx for 7d
Add fluclox if staph suspected.  1st: Trimethoprim 200mg BD
 1st: Nitrofurantoin 50mg QDS (avoid in ↓ eGFR)
Atyp Chlamydia: tetracycline  2nd: Cephalexin 500mg BD
PCP: Co-trimoxazole  2nd: Amoxicillin 500mg TDS
Legionella: Clarithro + rifampicin  Prostatitis: Cipro 500 mg BD for 28d

HAP
 Mild / <5d: Co-amoxiclav 625mg PO TDS for 7d Genital Tract
 Severe / >5d: Tazocin for 7d  Chlamydia / NSU: azithromycin 1g STAT
 ± vanc for MRSA  Gonorrhoea
 ± gent for Pseudomonas  Azithromycin 1g STAT + ceftriaxone 500mg IM
 PID: ofloxacin + metronidazole
Aspiration Pneumonia
 Co-amoxiclav 625mg PO TDS for 7d
GIT
Exacerbation of COPD  GI sepsis: cefuroxime + metronidazole
 Rx if ↑ sputum purulence c̄ ↑sputum volume or ↑  Campylobacter: ciprofloxacin
dyspnoea or consolidation on CXR  Shigella: ciprofloxacin
 Amoxicillin 500mg PO TDS for 7d  C. diff
 Or, doxy 200mg STAT + 100mg BD for 7d  1st: metronidazole PO
 2nd: Vancomycin PO
Legionella
 Levofloxacin or,
 Clarithromycin + rifampicin Sepsis
 Tazocin
PCP  ± Vanc if MRI suspected
st
 1 : co-trimoxazole  ± Gent for gram -ves
 2nd: pentamidine  If anaerobe: cef and met

TB Skin
 2mo: RHZE  Impetigo
 4mo: RH  Localised: topical fusidate
 Give pyridoxine 20mg OD throughout Rx  Widespread: fluclox 250 QDS
 Longer Rx if resistant organisms or extra-pulmonary TB  Erysipelas: Pen V 500mg QDS or ben pen
 Cellulitis
 Empiric: fluclox 500mg QDS
Infective Endocarditis  Known Strep: Pen V or Ben pen
 Empiric
 Acute severe: Fuclox + gent IV
 Subacute: Benpen + gent IV

 Streps: benpen + gent IV


 Enterococci: amoxicillin + gent IV
 Staphs: fluclox ± rifampicin IV
 Fungi: flucytosine IV + fluconazole PO.
 Amphotericin if flucytosine resistance or
Aspergillus.

© Alasdair Scott, 2012 58


Viral Infections
Herpes Viruses Influenza
 Bed rest + paracetamol
HSV  If severe
 Aciclovir  Mx in ITU
 Valaciclovir  Cipro and ao-amoxiclav: prevent Staph and Strep
 Famciclovir  Oseltamivir
 Neuraminidase inhibitor active vs. flu A and B
VZV  May be indicated if >1yr c̄ symptoms of <48hr
 Aciclovir  Zanamivir
 Inhaled NA inhibitor active vs. influenza A and B
CMV  >5yrs c̄ symptoms <48h
 Ganciclovir / valganciclovir
 Foscarnet
 Cidofovir HIV
Indications
Hepatitis  CD4 ≤350
 AIDS-defining illness
HBV  Pregnancy
 Combination or individual use of  HIVAN
 Peg-interferon-α  Co-infected c̄ HBV when Rx is indicated for HBV
 Tenofovir
 Entecavir Regimens
 1 NNRTI + 2 NRTIs
HCV  NNRTI = efavirenz
 Peg-interferon-α + ribavarin  NRTI = emtricitabine + tenofovir (Truvada)
 Atripla = efavirenz + emtricitabine + tenofovir
 PI + 2 NRTIs
 PI = lopinavir (+ low dose ritonavir = Kaletra)

Aim
 Undetectable VL after 4mo
 If VL remains high despite good compliance
 Change to a new drug combination
 Request resistance studies

© Alasdair Scott, 2012 59


Protozoal Infections Fungal Infections
Malaria PCP
 High-dose co-trimoxazole IV
Prophylaxis  Or, pentamidine IVI
 No resistance: proguanil + chloroquine  Prednisolone if severe hypoxaemia
 Resistance: mefloquine or malarone
 Malarone: atovaquone + proguanil
Cryptococcal Meningitis
Rx  Amphotericin B + flucytosine for 2wks
 Benign: chloroquine then primaquine  Then fluconazole for for 6mo / until CD4 >200
 Falciparum: riamet or malarone
 Riamet: artemether + lumefantrine
 IV antimalarials if severe
Candida
 Oral: nystatin drops
Toxoplasmaosis  Thrush: clotrimazole
 Pyrimethamine + sulfadiazine  Systemic: amphotericin B or voriconazole

Giardia Tinea Infections


 Tinidazole  Skin: terbinafine or topical ketoconazole / miconazole
 Scalp: griseofulvin or terbinafine
 Nails: terbinafine
Entamaeoba histolytica
 Metronidazole
Pityriasis versicolor
 Selenium or ketoconazole shampoo

Aspergillus
 Amphotercin B
 Itraconazole, voriconazole

© Alasdair Scott, 2012 60


Endocrine
Contents
Oral Hypoglycaemics................................................................................................................................................................... 62 
Insulin .......................................................................................................................................................................................... 63 
Thyroid Drugs .............................................................................................................................................................................. 64 
Pituitary Drugs ............................................................................................................................................................................. 64 
Calcium Metabolism .................................................................................................................................................................... 65 
Sex Hormones ............................................................................................................................................................................. 66 
T2DM ........................................................................................................................................................................................... 67 
Insulin Dependent DM ................................................................................................................................................................. 67 
Thyroid Disorders ........................................................................................................................................................................ 68 
Thyrotoxicosis.............................................................................................................................................................................. 68 
Adrenal Steroids .......................................................................................................................................................................... 68 
Pituitary Disorders ....................................................................................................................................................................... 69 
Osteoporosis ............................................................................................................................................................................... 69 

© Alasdair Scott, 2012 61


Oral Hypoglycaemics
Drug MOA Side Effects Contraindications Interactions Other
Metformin Biguanide Lactic acidosis Caution in renal or Renally excreted
Insulin sensitizer hepatic impairment - ↓ dose or avoid if ↓eGFR
- ↓ gluconeogenesis GI upset
- ↑ peripheral glucose use - anorexia → wt. loss Contrast media Cannot cause hypos
- ↓ LDL and VLDL General anaesthesia
Recent MI
Pioglitazone Thiazolidinedione Wt. gain H/L Don’t use with insulin
Peripheral insulin sensitizer Fluid retention V. protein bound
PPAR gamma ligand (nuclear receptor Hepatotoxicity Insulin use Hepatic metabolism
involved in glucose and lipid homeostasis) May exacerbate HF ACS
Monitor LFTs
Gliclazide (s) Sulfonylureas Hypoglycaemia Severe L/R fx ↑d by: Renally excreted
Tolbutamide (s) Insulin secretagogues - can be prolonged - sulphonamides
Glipizide (s) Acute porphyria - trimethoprim V. albumin bound
Glibenclamide (l) Block hyperpolarising K+ channel on β cells Wt. gain (↑ appetite) - NSAIDs
→ depolarisation and insulin release GI upset - warfarin Caution in elderly c̄ ↓ renal function
(Chlorpropamide) Headache - fibrates
Avoid long-acting (glibenclamide) in
elderly – use gliclazide or tolbutamide
Nateglinide Meglitinides Hypoglycaemia V. short acting → ↓ risk of hypo
Repaglinide Insulin secretagogues Give before meal
Block hyperpolarising K+ channel
Exenatide Insulin secretagogue Hypoglycaemia Admin by SC injection
Liraglutide GLP-1 analogue GI upset
- ↑ insulin secretion and sensitisation
Sitagliptin Insulin secretagogues Hypoglycaemia
Vildagliptin Dipeptidylpeptidase-4 inhibitor GI upset
DPP-4 breaks down endogenous GLP-1
Acarbose Intestinal α-glucosidase inhibitor Flatulence IBD Monitor LFTs
Delays carb absorption → ↓ post-prandial Loose stools/diarrhoea L
blood glucose Abdo pain / bloating
Little effect on fasting glucose Hepatotoxicity (rare)

© Alasdair Scott, 2012 62


Insulin
Short-acting Intermediate- and long- acting
Examples Soluble Insulin Insulin Analogues Isophane Insulin Insulin Analogues
Actrapid Aspart: NovoRapid Insulatard Glargine: Lantus
Humulin S Lispro: Humalog Humulin I Detemir: Levemir
Onset (sc) 30-60min 15min 1-3hrs
Peak (sc) 2-4hrs 1-3hrs 4-12hrs
Duration (sc) 6-8hrs 2-5hrs 12hrs Glargine: 24hrs
Detemir: 20hrs
Uses Maintenance: 15-30min before meals Glargine is given OD (nocte) as basal therapy
DKA Long acting analogues ↓ nocturnal hypoglycaemia
Sliding scales

Effects of insulin Side Effects


 Hypoglycaemia
Adipose tissue  At risk: EtOH binge, β-B (mask symptoms),
 ↑ lipoprotein lipase activity → ↓ TGs elderly
 ↑ GLUT-4 activity → ↑ glucose storage as fat  Need to admit sulfonylurea-induced hypo
 ↓ lipolysis → ↓ fatty acids release into circulation  Lipohypertrophy
 Rotate injection site: abdomen, thighs
Liver  Wt. gain in T2DM
 ↓ glycogenolysis  ↓ wt. gain if insulin given c̄ metformin
 ↓ gluconeogenesis
 ↑ glycogenesis Problems c̄ Actrapid in the Basal-Bolus Regime
 Inhibition of ketogenesis
 Absorbed over 3-4hrs: not physiological
 Must give 15min before meal
Muscle
 Can → late post prandial hypoglycaemia
 ↓ proteolysis
 Immediate post-prandial hyperglycaemia may → ↑ risk
 ↑ GLUT-4 activity → ↑ glucose uptake
of DM complications
NB. Lipoprotein lipase allows triglyceride uptake from LDL by
adipocytes.
Short Acting Insulin Analogues
 Fast onset and shorter duration
Factors Affecting Absorption  Can be given just before start of meal
 Temperature  Less chance of post-prandial hypoglycaemia
 Exercise
 Preparation: actrapid vs. glargine
 Dlvry method
 IV: 1min peak conc
 SC: 90min peak conc
 Injection site and depth

Administration
 s/c: typical route
 IVI
 DKA
 Control in critical illness
 Control in peri-operative period

© Alasdair Scott, 2012 63


Thyroid Drugs

Drug MOA Side Effects Contraindications Interactions Other


Thyroxine Precipitation of heart failure W+ Introduce T4 slowly in the elderly
Osteopaenia
AF
Over-Rx → hyperthyroidism
Carbimazole Thionamides Agranulocytosis Pregnancy Propythiouracil is reserved for those
Propythiouracil - thyroperoxidase inhibitors - often transient and benign Children intoleant of carbimazole due to risk of
- prevent iodination of tyrosine Tracheal compression hepatitis
- → ↓ T4/T3 synthesis Hypersensitvity: rash, pruritis - do surgery
Titrate to normal TSH or block and
Carbimazole is a pro-drug Hepatitis replace.
- converted to methimazole
Radioiodine: I131 Radioiodine is localised to thyroid → hypothyroidism Stop thionamide before use
→ destruction of gland Can precipitate thyroid storm

Pituitary Drugs

Drug MOA Side Effects Contraindications Interactions Other


Bromocriptine Ergot-derived Da agonists Fibrosis Cardiovascular disease Levels ↑d by: Use
Cabergoline GI upset: esp. nausea Porphyria - octreotide - prolactinoma
Pergolide Postural hypotension Psychosis - macrolides - can be used in acromegaly
Drowsiness
Neuropsych syndromes Monitor heart c̄ echo
Octreotide Somatostatin analogues Diarrhoea Use
Lanreotide Gallstones - acromegaly
- carcinoid syndrome
Pegvisomant GH receptor antagonist Use
- acromegaly
Metyrapone 11β-hydroxylase inhibitor Hypoadrenalism Use
- inhibits adrenal cortisol - can be used Cushing’s syn.
production - particularly if resistant to surgery

© Alasdair Scott, 2012 64


Calcium Metabolism

Drug MOA Side Effects Contraindications Interactions Other


Cinacalcet Calcimimetic Used for Rx of 2O HPT in ESRF
- → ↓ PTH secretion
Sevelamer Phosphate binder GI upset GI obstruction Used to ↓PO4 in ESRF
Lanthanum
Alendronate Bisphosphonates GI upset Achalasia Use
Etidronate ↓ ostoclastic bone resorption Oseophagitis and ulcers Oesophageal stricture - Prevention of osteoporotic #s
Pamidronate Osteonecrosis of the jaw - Prevention of osteoporosis
Ibandronate Diffuse MSK pain - Hypercalcaemia of malignancy
Risedronate - Paget’s
Zoledronate
Take c̄ glass of water on an empty
stomach 30min before breakfast,
staying upright.
Strontium ↑ bone formation DRESS syndrome Use if bisphosphonates not tolerated
↓ bone resorption - Drug Rash
- Eosinophilia
- Systemic Symptoms: fever, ↑wcc
Teriparetide Recombinant PTH GI upset Skeletal malignancies
- pulsatile admin → ↑ bone Paget’s
formation and ↓ resorption Severe R
Denosumab Anti-RANK ligand Used if bisphosphonates not tolerated
- ↓ osteoclast activation
Ergocalciferol Vitamin D2 Commonly used for Vit D deficiency
Colecalciferol Vitamin D3
Alfacalcidol 1α (OH) Vit D3 Use in renal disease
Calcitriol 1, 25, (OH) Vit D3 Use in renal disease

© Alasdair Scott, 2012 65


Sex Hormones

Drug Benefits Side Effects Contraindications Interactions Other


COCP Contraception ↑ risk of VTE Personal Hx of VTE P450 metabolism Don’t need extra
↓ dysmenorrhoea, ↓ menorrhagia Small ↑ risk of breast Ca Risk of VTE - ↓ effectiveness c̄ contraception when taking c̄
↓ PMT Small ↑ risk of IHD Risk of arterial disease enzyme inducers oral Abx that don’t induce
↓ benign breast disease Gallstones Hx of breast Ca liver enzymes, unless d/v
↓ ovarian and endometrial Ca Cholestatic jaundice ↑ fx of steroids
↓ risk of PID Breast tenderness
Hepatoma
POP n/v Severe arterial disease P450 metabolism
Headache Hx of breast Ca - ↓ effectiveness c̄
↑ wt. enzyme inducers
Breast tenderness
HRT ↓ hot flushes, ↓ vaginal dryness, ↑libido ↑ risk of Ca: breast, endometrial, ovarian Oestrogen-dependent Ca Excess Ca risk disappears w/i
↓ urinary frequency / urgency ↑ risk of VTE Hx of breast Ca 5yrs of stopping
↓ risk of bowel Ca ↑ risk of stroke and IHD UnDx vaginal bleeding
↓ risk of osteoporotic #s Cholestatic jaundice VTE

COCP Cautions and Contraindications


VTE Risk: caution if 1, avoid if ≥2
 FHx of VTE
 BMI >30, avoid if >35
 Long-term immobilisation
 Hx of superficial thrombophlebitis
 >35yrs, avoid if >50yrs
 Smoking

Arterial Risk: caution if 1, avoid if ≥2


 FHx of arterial disease
 DM
 HTN
 Smoking, avoid if >40/d
 >35yrs, avoid if >50yrs
 Migraine w/o aura, avoid if migraine c̄ aura

© Alasdair Scott, 2012 66


T2DM Insulin Dependent DM
1. Lifestyle Modification: DELAYS Principles
 Diet  Ensure pt. education about
 Exercise  Self-adjustment c̄ exercise and calories
 Lipids (statins if >40 regardless of lipids)  Titrate dose
 Aspirin (consider if >50yrs or <50 c̄ other RFs)  Family member can abort hypo c̄ sugary drinks
 ABP ↓ or GlucoGel
 Yearly / 6moly f/up  Pre-prandial BM don’t tell you how much glucose is
 Smoking cessation needed
 Fasting BM before meal informs re long-acting insulin
dose.
 Finger-prick BM after meal informs re short-acting
2. Start Metformin
insulin dose (for that last meal)
(if HBA1c >target after lifestyle changes)
 SE: nausea, diarrhoea, abdo pain, lactic acidosis
 CI: GFR<30, tissue hypoxia (sepsis, MI), morning
before GA and iodinated contrast media
Common Regimes
 500mg after evening meal, ↑ing to 2g max.
BD Biphasic Regime
 BD insulin mixture 30min before breakfast and dinner
 Rapid-acting: e.g. actrapid
3. Metformin + Sulfonylurea  Intermediate- / long-acting: e.g. insulatard
(if HBA1c >target)
 T2 or T1 DM c̄ regular lifestyle: children, older pts.
 E.g. gliclazide MR 30mg c̄ breakfast
 Assoc, c̄ fasting hyperglycaemia
 SE: hypoglycaemia, wt. gain
 CI: omit on morning of surgery Basal-Bolus Regime
 Bedtime long-acting (e.g. glargine) + short acting before
Other Options
each meal (e.g. lispro)
 Consider adding a rapid-acting insulin  Adjust dose according to meal size
secretagogue (e.g. nateglinide) to metformin  ~50% of insulin given as long-acting
instead of a sulfonylurea.
 T1DM allowing flexible lifestyle
 May be preferable if erratic lifestyle.
 Best outcome
 Consider adding pioglitazone to metformin
instead of a sulfonylurea
OD Long-Acting Before Bed
 Initial regime when switching from tablets in T2DM
4. Additional Therapy
Illness
st
1 line  Insulin requirements usually ↑ (even if food intake ↓)
 Add insulin → insulin + metformin + sulfonylurea  Maintain calories (e.g. milk)
 Check BMs ≥4hrly and test for ketonuria
2nd line  ↑ insulin dose if glucose rising
 Add sitagliptin or pioglitazone if insulin
unacceptable
 Employment, social or recreational issues
 Obesity
 Metformin + sulfonylurea + sitagliptin / pio

3rd line
 Add exenatide (SC) if insulin unacceptable or
BMI>35
 Metformin + sulfonylurea + exenatide

4th line
 Consider acarbose if unable to use other glucose-
lowering drugs

Targets
 Capillary blood glucose
 Fasting: 4.5-6.5mM
 2h post-prandial: 4.5-9mM
 HbA1c
 Reflects exposure over last 6-8wks
 Aim <45 - 50mM (6 - 7%)

© Alasdair Scott, 2012 67


Thyroid Disorders Adrenal Steroids
Hypothyroidism Glucocorticoid Replacement
 Need for 1O and 2O adrenal failure
Levothyroxine
 Titrate to normalize TSH Hydrocortisone
 Enzyme inducers ↑ thyroxine metabolism  Preferred due to gluco- and mineralo-corticoid activity.
 Clinical improvement takes ~2wks  20-30mg in divided doses
 10mg AM, 5mg lunch, 5mg evening
Caution
 In elderly pts. c̄ subclinical heart failure thyroxine can ↑ dose at intercurrent illness / injury
ppt. acute worsening: palpitations, angina, MI  If eating
 Introduce thyroxine gradually  3x normal dose for 3d
 Excessive thyroxine → osteopaenia and AF  Then 2x normal dose for 3d
 Vomiting / can’t tolerate PO
 100mg IV TDS

Thyrotoxicosis
Cushing’s
Confirm Aetiology  Treat underlying cause: e.g. pituitary or adrenal tumour
 Use Tc scan to determine if thyroxicosis is high or low  May use drugs temporarily or permanently if pt. can’t
uptake. undergo surgery: e.g. lung Ca
 Low
 Subacute de Quervain’s thyroiditis Metyrapone
 Postpartum thyroiditis  Inhibits final step in cortisol synthesis
 Amiodarone  80% response in Cushing’s disease
 High  Usually temporary
 Graves: 40-60%  Can be used as part of block and replace strategy
 TMNG: 30-50%
 Thyroid Adenoma: 5% Other anti-glucocorticoid drugs
 Functioning thyroid Ca  Ketoconazole: inhibits steroid synthesis
 Mifepristone: progesterone and glucocorticoid receptor
Mx antagonist
 Low Uptake
 Symptomatic: propranolol, atenolol Mineralocorticoid Replacement
 NSAIDs for de Quervain’s
 Need for 1O adrenal failure only
 High Uptake
 β-B Fludrocortisone
 Carbimazole
 Balance between HTN and postural hypotension
 Titration to normal TSH
 Or, block and replace
 Radioiodine
 Surgery
Primary Hyperaldosteronism
Bilateral adrenal hyperplasia (70%)
 Spironolactone
 Eplerenone
 Amiloride

Conn’s adenoma (30%)


 Surgical excision or medical therapy

© Alasdair Scott, 2012 68


Pituitary Disorders Osteoporosis
Cranial DI Conservative
 Desmopressin  Stop smoking, ↓ EtOH
 Nasal spray  Wt. bearing or balancing exercise (e.g. Tai Chi)
 IV  Ca and vit-D rich diet
 Avoid dilutional hyponatraemia by daily polyuria  Home-based fall-prevention program c̄ visual
episode assessment.

Acromegaly 1O and 2O prevention of osteoporotic #s


st
 1 line: trans-sphenoidal excision  Bisphosphonates: alendronate is 1st line
 2nd line: somatostatin analogues – octreotide  Ca and Vit D supplements
 3rd line: GH antagonist – pegvisomant  e.g Calcium D3 Forte
 4th line: radiotherapy  Strontium ranelate: bisphosphonate alternative

Somatostatin Analogues
 90% respond Alternative for 2O prevention of osteoporotic #s
 IGF1 normalised in 60%  Teriparetide: PTH analogue → new bone formation
 Denosumab: anti-RANKL → ↓ osteoclast activation
 Raloxifene: SERM, ↓ breast Ca risk cf. HRT
Prolactinoma
 1st line: D2 agonist
 2nd line: Trans-sphenoidal excision
 If visual or pressure symptoms don’t response to
medical Rx

D2 Agonists
 Bromocroptine, Cabergoline
 ↓ PRL secretion and ↓ tumour size

Hypopituitarism
 ACTH: hydrocortisone
 GH: rh-GH
 FSH / LH
 Testosterone
 OCP
 TSH: T4

© Alasdair Scott, 2012 69


Malignancy and
Immunosuppression
Contents
Anti-Proliferative Agents .............................................................................................................................................................. 71 
Inhibitors of Cell Signalling .......................................................................................................................................................... 71 
Chemotherapeutics ..................................................................................................................................................................... 72 
Immunosuppression .................................................................................................................................................................... 73 

© Alasdair Scott, 2012 70


Anti-Proliferative Agents

Drug MOA Side Effects Contraindications Interactions Use Other


Cyclophosphamide Alkylates DNA BM suppression Cancer Give mesna to prevent
Affects B cells > T cells Haemorrhagic cystits RA haemorrhagic cystitis
Alopecia SLE
Sterility: esp. males Systemic sclerosis Activated by P450
Wegener’s

Cisplatin Alkylates DNA BM suppression Cancer Carboplatin is assoc. c̄


Severe n/v - Ovarian less severe SEs
Nephrotoxic - Lung: SSLC
Ototoxic - Testicular Requires pre-admin
Peripheral neuropathy hydration
Azathioprine Blocks de novo purine BM suppression Allopurinol → ↑ toxicity Prevent Tx rejection Do TPMT (thiopurine
synthesis Hepatotoxicity Steroid-sparing agent methyltransferase) assay
Active metabolite is 6- n/v/d - IBD before use.
mercaptopurine Arthralgia - SLE
Affects T cells > B cells - RA 50% of pts intolerant of
azathioprine tolerate 6-MP
Mycophenolate Blocks de novo nucleotide BM suppression Prevent Tx rejection
mofetil synthesis Skin malignancy AI disease
Affects T cells > B cells GI upset

Methotrexate Dihydrofolate reductase BM suppression R ↑ toxicity c̄ Cancer Give c̄ folinic acid to ↓


inhibitor Pulmonary fibrosis L - NSAIDs RA risk of myelosuppression
Hepatotoxic - ciclosporin Psoriasis
Mucositis - steroids Crohn’s Monitor U+E, FBC, LFT
Chlorambucil Alkylates DNA BM suppression Cancer: e.g. CLL
EM → SJS

Inhibitors of Cell Signalling

Drug MOA Side Effects Contraindications Interactions Use Other


Ciclosporin Calcineurin inhibitors Nephrotoxic P450 metabolism Prevent Tx rejection Monitor LFTs
- Blocks IL-2 production Hepatic dysfunction GvHD
Tremor UC
Hypertrichosis RA
Gingival hypertrophy Psoriasis
Encephalopathy
Tacrolimus Nephrotoxic (< cf. ciclosporin) P450 metabolism Prevent Tx rejection
Diabetogenic
Neurotoxic (> cf. ciclosporin)
Sirolimus Blocks mTOR pathway Dyslipidaemia Prevent Tx rejection
© Alasdair Scott, 2012 71
Chemotherapeutics
Important Cytotoxic Classes Specific Problems

Alkylating agents Drug Problems


 Cyclophosphamide, chlorambucil, busulfan, cisplatin Cyclophosphamide Haemorrhagic cystitis: give mesna
 Mechanism Hair loss
 DNA x-linking BM suppression
 Base mis-paring Doxorubicin + other Cardiomyopathy
 Excision of alkylated DNA → strand breaks anthracyclines Extravasation reactions
Bleomycin Pulmonary fibrosis
Antimetabolites: methotrexate, 5-FU Vincristine Peripheral neuropathy
- don’t give IT
Cytotoxic Abx Cisplatin Severe n/v
 Anthracyclines: doxorubicin, daunorubicin Nephrotoxic
 Bleomycin Ototoxic
 Mechanism Peripheral neuropathy
 Intercalate c̄ DNA Paclitaxel Peripheral neuropathy
 Free radical formation Hypersensitivity
- Pre-Rx c̄ antihistamines + steroids
Microtubule Inhibitors 5-FU Palmar-plantar erythrodysthesia
 Vinca alkaloids: vincristine, vinblastine Mucositis
 Taxanes: paclitaxel

Topoisomerase Inhibitors Mx Chemo-induce Emesis


 Etoposide
Low risk of emesis
 Domperidone / metoclopramide started pre-Rx
Immune Modulators: thalidomide, lenalidomide
High risk of emesis
MAbs  Ondansetron / granisetron +
 Trastuzumab (anti-Her2): breast Ca  Dexamthasone +
 Bevacizumab (anti-VEGF): RCC, CRC, lung  Aprepitant
 Cetuximab (anti-EGFR): CRC
 Rituximab (anti-CD20): NHL Common Regimens
Tyrosine Kinase Inhibitors Breast Ca: FEC
 Erlotinib: lung Ca  5-FU
 Imatinib: CML  Epirubicin
 Sunitinib: RCC  Cyclophosphamide
Endocrine Modulators: tamoxifen, anastrazole Breast Ca: CMF
 Cyclophosphamide
 Methotrexate
Common Side Effects  5-FU
 n/v: prophylactic anti-emetics
 Alopecia Testicular Teratoma: BEP
 Neutropenia: 10-14d after chemo  Bleomycin
 Extravasation of chemo agent  Etoposide
 Pain, burring, bruising @ infusion site  CisPlatin
 Stop infusion, give steroids, apply cold pack
 Liaise early c̄ plastics Ovarian
 Hyperuricaemia  Carboplatin
 Oral mucositis  Paclitaxel

NHL: R-CHOP
 Rituximab
 Cyclophosphamide
 Hydroxydaunomycin (doxorubicin)
 Oncovin
 Prednisolone

HL: ABVD
 Adriamycin (doxorubicin)
 Bleomycin
 Vinblastine
 Dacarbazine
© Alasdair Scott, 2012 72
Immunosuppression
Prednisolone Transplant Regimens
MOA Liver
 Inhibits PLA2 → ↓ PG and ↓ PAF  Tacrolimus
 ↓ PMN extravasation → ↑ PMN in blood  Azathioprine
 Lymphocyte sequestration in tissues → lymphopenia  Prednisolone ± withdrawal @ 3mo
 ↓ Phagocytosis
 Lymphocyte apoptosis Renal
 ↓ Ab production  Pre-op induction
 ↓ cytokine and proteolytic enzyme release  Alemtuzumab (Campath: anti-CD52)
 Post-op
Dose  Predniolone 7d
 Use lowest possible dose: alternate days if possible  Tacrolimus long-term
 Graded withdrawal if used >3wks

Advice
 Don’t stop steroids suddenly
 Consult doctor when unwell
 ↑ dose c̄ illness or stress (e.g. pre-op)
 Carry a steroid card: dose and indication
 Avoid OTCs: e.g. NSAIDs
 Osteoporosis and PUD prophylaxis
 Ca and vitamin D supplements: Calcichew-D3
 Bisphosphonates: alendronate
 PPI: lansoprazole

SEs (mostly long-term use >6wks)

 GI Candidiasis
PUD
Oesophageal ulceration
Pancreatitis

 Cardio HTN
CCF

 MSK Proximal myopathy


Osteoporosis

 Endo Growth suppression


HPA suppression
Cushing’s syndrome

 Metabolic Na and fluid retention


↑PMN
↓K

 CNS Depression, psychosis

 Eye Cataracts
Glaucoma

 Immune ↑ susceptibility to infection

Interactions
 Fx ↓d by hepatic inducers
 Fx ↑d by
 Hepatic inhibitors
 OCP

© Alasdair Scott, 2012 73


Musculoskeletal
Contents
Rheumatic Disease ..................................................................................................................................................................... 75 
Anti-Gout ..................................................................................................................................................................................... 75 
NSAIDs ........................................................................................................................................................................................ 76 
Neuromuscular Drugs ................................................................................................................................................................. 77 
Osteoarthritis ............................................................................................................................................................................... 78 
Rheumatoid Arthritis .................................................................................................................................................................... 78 
Gout ............................................................................................................................................................................................. 79 

© Alasdair Scott, 2012 74


Rheumatic Disease

Drug MOA Side Effects Contraindications Interactions Other


Sulfasalazine 5-Aminosalicylate Sulfasalazine has ↑ SEs Caution in renal or Monitor FBC
Mesalazine - blood dyscrasias hepatic impairment
Unknown MOA - hepatitis Better for use in young women
- rash, urticaria cf. methotrexate
- oligospermia
- pulmonary fibrosis
Methotrexate Dihydrofolate reductase inhibitor BM suppression R ↑ toxicity c̄ Cancer
Pulmonary fibrosis L - NSAIDs RA
Hepatotoxic - ciclosporin Psoriasis
- steroids Crohn’s
Hydroxychloroquine ↓ activation of dendritic cells Visual change Caution in G6PD Monitor vision
- rarely retinopathy deficiency
Seizures
BM suppression
Penicillamine Nephrotic syndrome SLE Chelates Cu and Pb
Drug-induced lupus Prevents stones in cystinuria
Taste change
Infliximab Chimeric anti-TNF mAb Severe infections TB Screen for TB before use
- Remicade TB
Allergic reactions Parenteral admin
CCF
CNS demyelination Give c̄ hydrocortisone to ↓
↑ AI disease and C allergic SEs

Anti-Gout

Drug MOA Side Effects Contraindications Interactions Other


Colchicine Diarrhoea Caution in renal impairment
Renal impairment
Allopurinol XO inhibitor Severe skin reactions Caution in R and L ↓ metabolism of azathioprine Initial Rx can → ↑ gout
- EM → SJS - ↓ dose - AVOID - initiate c̄ NSAID / coplchicine cover
GI upset
Hepatotoxic
Feboxustat XO inhibitor Headache Caution in R and L
Rash
Abnormal LFTs
Probenecid Uricosuric GI upset Renal impairment
Rasburicase Recombinant uric
oxidase
© Alasdair Scott, 2012 75
NSAIDs

Drug MOA Side Effects Contraindications Interactions Other


Least → Most Toxic Non-selective COX inhibitors Gastritis and PUD Renal or cardiac failure ↑ bleeding c̄ warfarin Can be given c̄ other agents
- Ibuprofen ↓ GFR PUD ↓s effects of ACEis and ARBs for gastro protection
- Diclofenac Analgesic Interstitial nephritis Severe hepatic impairment ↑ toxicity of methotrexate - PPI
- Aspirin Antipyretic Papillary necrosis - H2RAs
- Naproxen Anti-inflammatory ↑K Caution - mioprostol
- Indomethacin Peripheral oedema - in the elderly
- Ketoprofen Bronchospasm - asthma
Hypersensitivity
- EM → SJS
Celecoxib Selective COX-2 inhibitor ↑ cardiovascular IHD ↓s effects of ACEis and ARBs Assess CV risk before use
events Cerebrovascular disease ↑ toxicity of methotrexate
Renal SEs as above L/R failure Only used for short periods
in young pts. c̄ intolerance
of other NSAIDs

Eicosanoid Synthesis

© Alasdair Scott, 2012 76


Neuromuscular Drugs

Drug MOA Side Effects Contraindications Interactions Other


Pyridostigmine Anticholinesterases Cholinergic Asthma Edrophonium preferred for the Dx of MG
Neostigmine - ↑ ACh in the synaptic cleft Intestinal / urinary obstruction due to its v. short t½.
Edrophonium - Enhance neuromuscular transmission
Pyridostigmine preferred for the Rx of
MG due to long t½
Baclofen GABAB agonist Sedation PUD fx ↑d by TCAs Rx painful muscle spasms
↓ tone
Skeletal muscle relaxant Nausea Don’t withdraw abruptly
Urinary disturbance - hyperthermia
- ↑ spasticity
Dantrolene Prevents Ca2+ release from Hepatotoxicity Hepatic impairment Used to relieve chronic spasticity and in
sarcoplasmic reticulum GI upset malignant hyperthermia

Skeletal muscle relaxant

© Alasdair Scott, 2012 77


Osteoarthritis Rheumatoid Arthritis
Conservative Conservative
 ↓ wt.  Refer to rheumatologist
 Alter activities: ↑ rest, ↓ sport  Regular exercise
 Physio: muscle strengthening  PT
 Walking aids, supportive footwear, home mods  OT: aids, splints

Medical Medical
 Analgesia  DAS28: monitor disease activity
 Paracetamol  DMARDs and biologicals: use early
 NSAIDs  Steroids: PO or intra-articular for exacerbations
 1st: Ibuprofen: 400mg TDS  NSAIDs: good for symptom relief
 2nd: diclofenac  e.g. arthrotec: but → diarrhoea
 Tramol  Mx CV risk: RA accelerates atherosclerosis
 Joint injection: local anaesthetic and steroids  Prevent osteoporosis and gastric ulcers

Surgical DMARDs
 Arthroscopic washout: esp. knee.
 1st line for treating RA
 Trim cartilage, remove foreign bodies.
 Early DMARD use assoc. c̄ better long-term outcome
 Arthroplasty: Replacement (or excision)
 Osteotomy: small area of bone cut out.  All DMARDs can → myelosuppression → pancytopenia
 Arthrodesis: last resort for pain management
Main agents:
 Novel Techniques
 Methotrexate: hepatotoxic, pulm. fibrosis, teratogenic
 Microfracture: stem cell release → fibro-cartilage
formation  Sulfasalazine: SJS, ↓ sperm count, pulmonary fibrosis
 Autologous chondrocyte implantation  Hydroxychloroquine: visual change, rash, seizures

Other Agents:
 Leflunomide: ↑ risk of infection and malignancy
 Gold: nephrotic syndrome
 Penicillamine: drug-induced lupus, taste change,
nephrotic syn.

Biologicals
Anti-TNF
 Severe RA not responding to DMARDs
 DAS28 >5.1
 Failed methotrexate + ≥1 other DMARD
 Screen and Rx TB first
 Infliximab: chimeric anti-TNF Ab
 Etanercept: TNF-receptor
 Adalimumab: human anti-TNF Ab
 SEs: ↑ infection (sepsis, TB), ↑ AI disease, ↑ Ca

Rituximab (anti-CD20 mAb)


 Severe RA not responding to anti-TNF therapy

Surgical
 Ulna stylectomy
 Joint prosthesis

© Alasdair Scott, 2012 78


Gout
Acute Rx
 NSAID: diclofenac or indomethacin
 Colchicine
 NSAIDs CI: warfarin, ulcers, heart HF, CRF
 SE: diarrhoea
 In renal impairment: NSAIDs and colchicine are CI
 → Use steroids

Prevention
 Conservative
 Lose wt.
 Avoid prolonged fasts and EtOH excess
 Xanthine Oxidase Inhibitors: Allopurinol
 Use if recurrent attacks, tophi or renal stones
 Introduce c̄ NSAID or colchicine cover for 3/12
 SE: rash, fever, ↓WCC (c̄ azathioprine)
 Use febuxostat (XO inhibitor) if hypersensitivity
 Uricosuric drugs: e.g. probenecid, losartan
 Rarely used
 Recombinant urate oxidase: rasburicase
 May be used pre-cytotoxic therapy

© Alasdair Scott, 2012 79


Emergencies
Contents
Cardiovascular............................................................................................................................................................................. 81 
Neurological................................................................................................................................................................................. 82 
Respiratory .................................................................................................................................................................................. 82 
Endocrine .................................................................................................................................................................................... 83 
Metabolic ..................................................................................................................................................................................... 84 

© Alasdair Scott, 2012 80


Cardiovascular
ACS Malignant HTN
 Oxygen
 Analgesia BP > 220 / 120 + either
 Morphine 5-10mg IV  Grade 3: haemorrhages, exudates
 Metoclopramide 10mg IV  Grade 4: papilloedema
 Anti-platelet
 Aspirin 300mg PO Mx
 Clopidogrel 300mg PO  Aim to ↓ DBP to 100-115mmHg over 4-6hrs
 Anti-ischaemia  Nitroprusside or labetalol
 GTN: 2 puffs or 1 tab SL  Phentolamine if phaeo known or suspected
 IV if pain continues
 Atenolol SVT
 STEMI: 5mg IV
 NSTEMI: 50mg PO /24h Unstable → DC cardioversion
 Anti-coagulate
 Enoxaparin Irregular Rhythm: Rx as AF

STEMI Regular Rhythm


st
 1 : PCI  Adenosine 6mg IV bolus, then 12mg, then 12mg
 2nd: thrombolysis  Choose from
 Add tirofiban in high-risk pts.  Verapamil
 Atenolol
NSTEMI  Digoxin
 Tirofiban  Amiodarone
 Angiography ± PCI w/i 96hrs
Broad Complex Tachycardia
Long-Term
 ACEi: e.g. lisinopril 2.5mg OD Unstable → DC cardioversion
 Aspirin 75mg OD
 β-B: e.g. bisoprolol 1.25mg OD Regular Rhythm
 Verapamil if β-B not tolerated  Amiodarone or lignocaine
 Clopidogrel 75mg OD
 STEMI: 1mo Irregular Rhythm
 NSTEMI: 1yr  Pre-excited AF: amiodarone or flecainide
 Statin: e.g. atorvastatin 80mg OD  TDP: MgSO4

Septic Shock
Acute Pulmonary Oedema
Initial Rx
Initial Mx  Oxygen
 Oxygen  Abx: cultures 1st then follow guidelines (e.g Tazocin)
 Diamorphine 2.5-5mg + metoclopramide  Fluids: 1L crystalloid or 500ml colloid over 30min
 Frusemide 40-80mg IV  If BP still ↓ consider CVP and further fluids
 GTN 2 puffs or 2 tabs SL  Aim CVP ≥8mmHg and UO >0.5ml/kg/hr
 ISMN 2-10mg/h IVI  Inotropes if SBP <90mmHg after fluid resus
 Keep SBP >90mmHg  Norad 1-10ug/min: maintain MAP >65mmHg

Rx underlying cause Further Mx


 Insulin sliding scale: keep glucose <8.3mM
If no improvement  DVT prophylaxis: enoxaparin 40mg OD SC
 CPAP  Stress ulcer prophylaxis: PPI
 If SBP <90: dobutamine
 2-20ug/kg/min IVI Cardiac Arrest
 Via a central line  Shockable
 Adrenaline 1mg + amiodarone 300mg after 3rd
Anaphylaxis shock
 Oxygen  Repeat adrenaline every other cycle
 Adrenaline 0.5mg IM: 0.5ml of 1:1000  Non-Shockable
 Repeat every 5min as necessary, guided by  Adrenaline 1mg as soon as IV access obtained
cardiorespiratory function  Repeat adrenaline every other cycle
 Chlorphenamine 10mg IV
 Hydrocortisone 200mg IV Bradycardia <40bpm
 Nebs: salbutamol 5mg + ipratropium 0.5mg  Atropine 0.6–1.2g (max 3g) IV
 Isoprenaline IVI

© Alasdair Scott, 2012 81


Neurological Respiratory
Meningitis Acute Asthma
Acute Mx Initial Rx
 Septicaemic  O2 driven nebulisers
 Ceftriaxone 2g IVI  Salbutamol 5mg
 + ampicillin 2g IVI /4h if >50yrs  Ipratropium 0.5mg
 Meningitic  Repeat salbutamol every 15min until
 Ceftriaxone ± ampicillin post-LP improvement
 Dexamethasone 0.15mg/kg IVI QDS  Monitor ECG
 Steroids
Continuing Mx  100mg hydrocortisone
 Ceftriaxone 2g IVI BD  Or, 50mg prednisolone

No Improvement
Status Epilepticus  MgSO4 2g IVI over 20min
 Salbutamol 3-20 ug/min IVI
Reverse Potential Causes  Aminophylline
 Thiamine 250mg IV if EtOH  If not already on theophylline
 100ml 20% glucose unless known to be normal  Load then IVI

IV Bolus Phase
 Lorazepam 2-4mg IV over 30s Acute COPD
 Repeat if no response w/i 2min
 Alternatives Initial Rx
 Diazepam 10mg IV/PR (20mg max)  24% O2
 Midazolam 10mg buccal  Blue venturi mask
 Aim for 88-92% SpO2
IV Infusion Phase  Hydrocortisone 200mg IV
 Phenytoin 18mg/kg IVI  Doxy 200mg PO STAT if evidence of infection
 50mg/min max
 Or, diazepam IVI No Improvement
 Consider aminophylline
Induction Phase  BiPAP
 Propofol or thiopentone

PE
 Oxygen
 Morphine + metoclopramide
 Massive PE: alteplase 50mg bolus STAT
 Stable: anticoagulate
 Enoxaparin 1.5mg/kg/24hr SC
 Warfarin: 5mg PO

© Alasdair Scott, 2012 82


Endocrine
DKA Thyroid Storm
 Fluid resuscitation + NGT
Dx  Bloods: TFTs + cultures if infection suspected
 Acidosis (↑AG): pH <7.3 (± HCO3 <15mM)  Propranolol PO/IV
 Hyperglycaemia: ≥11.1mM (or known DM)  Digoxin + LMWH if AF
 Ketonaemia: ≥3mM (≥2+ on dipstix)  Carbimazole then Lugol’s Iodine 4h later to inhibit
thyroid
Fluids  Hydrocortisone 100mg QDS IV
 0.9% NS infusion via large-bore cannula  Rx cause
 SBP<90 → 1L stat + more until SBP >90
 SBP>90 → 1L over 1h
 Then: 1L over next 2h, 1L/2h, 1L/4h, 1L/4h, 1L/6h
Myxoedema Coma
 Switch to 10% dex 1L/8h when glucose <14mM
 Correct any hypoglycaemia
 T3/T4 IV slowly (may ppt. myocardial ischaemia)
Insulin Infusion
 0.1u/kg/h Actrapid (6u if no wt., max 15u)  Hydrocortisone 100mg IV
 Transfer to sliding scale when resolved  Rx hypothermia and heart failure
 Ketones <0.3mM
 Venous pH >7.3 (HCO3 >18mM)
 Transfer to SC insulin when eating Addisonian Crisis
 Check CBG: glucose may be needed
nd
Start Potassium Replacement in 2 Bag of Fluids  Hydrocortisone 100mg IV 6hrly
 >5.5mM → Nil  IV crystalloid
 3.5-5.5mM → 40mM  Rx underlying cause
 <3.5mM → consult senior for review

Additional Therapy Phaeochromocytoma Emergencies


 LMWH  1st α-block
 Consider NaHCO3 if pH <6.9  Phentolamine: repeat to safe BP
 Find and Rx any infection  Phenoxybenzamine: exchange for phentolamine
 Longer t½
 2nd β-block
HONK  Only after α-blockade
 Avoids unopposed α-adrenergic stimulation
 Rehydrate c̄ 0.9% NS over 48h
 Surgery
 May need ~9L
 Electively after 406wks to allow full α-blockade
 Wait 1h before starting insulin and volume expansion
 It may not be needed  Phenoxybenzamine ↑d until significant postural
 Start low to avoid rapid changes in osmolality hypotension.
 e.g. 1-3u/hr
 LMWH

Hypoglycaemia
Alert and Orientated: Oral Carb
 Rapid acting: lucozade
 Long acting: toast, sandwich

Drowsy / confused but swallow intact: Buccal Carb


 Hypostop / Glucogel
 Consider IV access

Unconscious or Concerned re Swallow: IV dextrose


 50ml 50% or 100ml 20% glucose

Deteriorating / refractory / insulin-induced / no access


 1mg glucagon IM/SC
 Won’t work in drunks + short duration of effect (20min)
 Insulin release may → rebound hypoglycaemia

© Alasdair Scott, 2012 83


Metabolic
Hyperkalaemia Acute Poisoning
ECG Features (in order) Poison Rx
 Peaked T waves Aspirin Alkalinise urine: NaHCO3 + KCl
 Flattened P waves Benzodiazepine Flumazenil
 ↑ PR interval β-B Atropine, glucagon
 Widened QRS Cyanide Dicobalt edentate
 Sine-wave pattern → VF CO Hyperbaric O2
Digoxin Digibind
Mx Ethylene Glycol Ethanol
 Calcium gluconate 10ml 10% Heparin Protamine
 50ml 50% glucose + 10u insulin (Actrapid) Iron Desferrioxamine
 Salbutamol 5mg nebulizer Lead Sodium calcium edentate
 Calcium resonium 15g PO or 30g PR Methanol Ethanol
 Haemofiltration (usually needed if anuric) Opiates Naloxone
Organophosphates Atropine + pralidoxime
Paracetamol NAC
Hypokalaemia TCA NaHCO3 + O2
Warfarin: major bleed PCC + Vit K IV
ECG Features
 Result from delayed ventricular repolarisation
 Flattened / inverted T waves
 Prominent U waves (after T waves)
 ST depression
 Long PR interval
 Long QT interval

Mild: K >2.5
 Oral K supplements
 ≥80mmol/d

Severe: K <2.5 and/or dangerous symptoms


 IV K cautiously
 10mmol/h (Max 20mmol/h)
 Best to give centrally (burning sensation)
 Max central conc: 60mM
 Max peripheral conc: 40mM

Hypercalcaemia
Rehydrate
 1L 0.9% NS / 4h
 Monitor pts. hydration state

Frusemide
 Only start once pt. is volume replete
 Calciuric + makes room for more fluids

Bisphosphonates
 Ca bisphosphonate can’t be resorbed by osteoclasts
 Used to prevent recurrence
 Can obscure Dx as → ↓Ca, ↓PO4 and ↑PTH
 E.g. Pamidronate, Zoledronate (IV)

© Alasdair Scott, 2012 84


Revision
Contents
Prescribing Scenarios ................................................................................................................................................................. 86 
Prescriptions for Comment .......................................................................................................................................................... 88 
Useful Drugs to Know .................................................................................................................................................................. 90 

© Alasdair Scott, 2012 85


Prescribing Scenarios
Scenario 1 Scenario 4
68 year old man with T2DM and hypertension is brought to the A 59-year-old woman presents with a several year history of
A&E department with AF which started less than 2 hours increasing shortness of breath and a productive cough. She
earlier. His ventricular rate is about 130/min. DC cardioversion has a smoking history of at least 30 pack years but no other
is unsuccessful and he remains in hospital. He says he would medical history. On examination of the chest there is
prefer not to have a further attempt at the procedure. BP is widespread wheeze and coarse crackles cleared by coughing,
consistently higher that 150/90 during the daytime. but the sputum is very viscous though clear. Spirometry
Electrolytes, creatinine, and thyroid function are normal. confirms a mixture of obstructive and restrictive abnormalities.
Fasting blood glucose is 7.9, fasting total cholesterol is 6.6,
HDL cholesterol is 0.96 and TGs 3.4 Answer
 Formoterol inhaler 12 micrograms x2 daily + extra
Answer doses for symptom relief (maximum total dose 48
 Metformin 500mg x1 daily c̄ breakfast micrograms daily)
 Simvastatin 20mg x1 nocte  Carbocysteine 750mg x3 daily
 Verapamil 80mg x3 daily
 Lisinopril 10mg x1 daily
 Warfarin 5mg x1 daily Scenario 5
 Check INR on day 5 and adjust dose according
to Tait Regimen, aiming for an INR of 2.5. 73 year old woman is admitted with congestive heart failure.
She has pitting oedema to the umbilicus and is almost
immobile. Her BMI is 28 and BP is 121/65. On admission she
Scenario 2 is found to have a UTI, bacteriology results are awaited. Other
investigations show creatinine 133, fasting Glucose 5.1, total
72 year old woman is admitted with a fractured neck of femur cholesterol 4.8, HDL cholesterol 1.1. CXR confirms pulmonary
following a fall. She has successful surgery but despite congestion.
appropriate prophylaxis develops a DVT in one calf. While she
is being treated for this, she undergoes bone densitometry Answer
which reveals vertebral and femoral osteoporosis. She also  Oxygen 4L via nasal cannula: aim for SpO2 94-98%
has a lower UTI.  Frusemide 40 mg IV x1 daily
 Lisinopril 2.5mg x1 daily
Answer  Bisoprolol 1.25mg x1 daily
 Paracetamol 1g x4 daily  Simvastatin 20mg x1 nocte
 Codeine phosphate 30mg every 4 hours as required  Aspirin 75mg x1 daily
 Maximum daily dose 240mg  Cephalexin 250mg x4 daily for 7 days
 Enoxaparin 1.5mg/kg x1 daily SC until INR = 2.5  Enoxaparin 40mg x1 daily SC until mobile
 Warfarin 5mg x1 daily
 Check INR on day 5 and adjust dose according
to Tait regimen, aiming for an INR of 2.5.
 Continue warfarin for 6 weeks – 3 months.
 Alendronate 70mg x1 weekly
 Trimethoprim 200mg x2 daily for 7d

Scenario 3
A 62-year-old man is admitted with a 3-day-history of
increasing shortness of breath. On examination his
temperature is 37.7 C, he has right lower lobe consolidation
and also pitting oedema to his knees and JVP +6 cm. He is in
sinus rhythm at 88 bpm and his BP is 133/76 mm Hg and
oxygen saturation is 92%.

Answer
 Oxygen 4L via nasal cannula: aim for SpO2 94-98%
 Amoxicillin 500mg x3 daily for 7d
 Clarithromycin 500mg x2 daily for 7d
 Frusemide 40mg x1 daily

© Alasdair Scott, 2012 86


Scenario 6
A 73-year-old woman is admitted with a fractured neck of
femur following a fall. She had been lying on the floor of her
flat for at least 24 hours and was found to have a calf deep
vein thrombosis and a lower urinary tract infection. There were
no other abnormalities found on admission and she was not
known to have any other illnesses before her accident. She
has moderately severe pain and surgery is planned but for the
moment she is not “ nil by mouth”.

Answer
 Paracetamol 1g x4 daily
 Tramadol 50mg every 4 hours as required
 Maximum daily dose 300mg
 Enoxaparin 1.5mg/kg x1 daily SC
 No warfarin before surgery
 Trimethoprim 200mg x2 daily for 7d

Scenario 7
A 62-year-old man with type 2 diabetes and hypertension is
admitted with a 3-day history of increasing cough and
shortness of breath, and a temperature of up to 38.2°C. His
BMI is approximately 31kg/m2. On examination there are
signs of right lower lobe consolidation. Blood pressure varies
between 144/91 and 163/103 mm Hg. Electrolytes and
creatinine are within the normal range. Random blood
glucose is 14. 3 mmol/L, fasting total cholesterol is 6.1
mmol/L, HDL cholesterol 0.96 mmol/L and triglycerides 3.4
mmol/l.

Answer
 Oxygen 4L via nasal cannula: aim for SpO2 94-98%
 Amoxicillin 500mg x3 daily for 7d
 Clarithromycin 500mg x2 daily for 7d
 Lisinopril 2.5mg x1 daily
 Nifedipine MR 20mg x1 daily
 Metformin 500mg x1 daily c̄ breakfast
 Simvastatin 20mg x1 nocte

© Alasdair Scott, 2012 87


Prescriptions for Comment
Prescription 1 Prescription 2
69 year old man has COPD with heart failure. He also has 68 year old woman has PD for at least 5 years. She suffers a
BPH with obstructive symptoms. He is admitted with an TIA affecting her speech and the right hand side of her face
infective exacerbation of the COPD. lasting about 6 hours. BP is 158/62 seated and 132/54
standing, in sinus rhythm. Total cholesterol is 7.8, HDL 1.2.
Bendroflumethazide 5mg OD She is not diabetic and her renal and liver function are normal.
Bisoprolol 5mg OD There is 60% stenosis of the left internal carotid and 15% of
Ciprofloxacin 500 mg BD the right.
Doxazosin 8mg OD
Enalapril 5mg OD Aspirin 75mg OD
Ipratropium Bromide 40 ug QDS inhaled Atenolol 50mg OD
Clopidogrel 75mg OD
Comments Co-careldopa 250/25 BD
 Bendroflumethazide Pravastatin 20mg OD
 Not the preferred diuretic in heart failure. Switch
to frusemide. Comments
 Bisoprolol  Aspirin and Clopidogrel
 High dose for heart failure. Titrate up slowly from  TIA occurred despite aspirin and clopidogrel. The
1.25mg OD. MATCH trial found that the combination of aspirin
 COPD was considered a relative contraindication and clopidogrel was not beneficial in
in COPD, but this is now controversial. A recent neurovascular disease and resulted in increased
trial in the BMJ suggested that cardioselective β- bleeding risk. Stop the clopidogrel and add
blockers may be beneficial. dipyridamole MR. The NASCET and ECST trials
 Ciprofloxacin demonstrated benefit of carotid endarterectomy
 Not the preferred choice for acute exacerbation in symptomatic patients with 50-69% stenosis if
of COPD. Switch to amoxicillin. the operative risk is low. This option should be
 Doxazosin considered.
 High dose for treatment of BPH. Either reduce to  Atenolol
2-4mg x1 daily or switch to tamsulosin which may  Not the first line drug for the treatment of
be more effective. hypertension in patients over 55 years. In
 Enalapril addition she demonstrates a significant postural
 Dose is low. Gradually increase to 10-20mg BD if drop. Switch her antihypertensive to nifedipine
tolerated. MR 20mg x1 daily.
 Ipratropium bromide  Pravastatin
 May exacerbate BPH. Switch to formoterol  Cholesterol is high despite treatment c̄
inhaler. pravastatin. Pravastatin has a relatively low
potency and the dose is low. Switch to
simvastatin 20mg x1 nocte.
 Co-careldopa
 Dose is expressed incorrectly: dose of carbidopa
should be given first: i.e. 25/250 x2 daily.
 Dose is quite high and not well spaced out (L-
DOPA has a short t½). In addition she has
postural hypotension. Change regimen to 25/100
x3-4 daily.

© Alasdair Scott, 2012 88


Prescription 3 Prescription 4
A 65-year-old man has COPD and permanent atrial fibrillation A 72-year-old woman has Parkinson’s disease and develops
with mild heart failure. He is admitted with an infective nausea on initiation of treatment. She has had isolated
exacerbation of the COPD. systolic hypertension for at least 5 years. A few weeks later
she develops distressing visual hallucinations. Her blood
Verapamil modified release 120 mg x1 daily pressure is 117/62 mm hg seated and 98/55 mm Hg standing.
Bisoprolol 5 mg x1 daily
Clarithromycin 500 mg x 2 daily Co-careldopa 250/25 1 tablet x 4 daily
Warfarin 4 mg x 1 daily (INR 2.8) Metoclopramide 10 mg x 3 daily
Lisinopril 20 mg x1 daily Prochlorperazine 5 mg x 1 at bedtime
Furosemide 40 mg x1 daily Irbesartan 150 mg x 1 daily

Answer Answer
 Bisoprolol  Co-careldopa
 High dose for heart failure. Titrate up slowly from  Dose is expressed incorrectly: dose of carbidopa
1.25mg OD. should be given first: i.e. 25/250 x2 daily.
 COPD was considered a relative contraindication  Dose is quite high. In addition she is experiencing
in COPD, but this is now controversial. A recent hallucinations and has postural hypotension.
trial in the BMJ suggested that cardioselective β- Reduce dose: e.g. 25/100 x3-4 daily.
blockers may be beneficial.  Irbesartan
 β-B + verapamil contraindicated  Dose is high and isn’t first line antihypertensive in
 Clarithromycin + Warfarin this age. May not even be necessary at all given
 Clarithromycin is a Cyp450 inhibitor. The dose of her blood pressure.
warfarin may need to be reduced  Metoclopramide and prochlorperazine
 Verapamil + Furosemide + Lisinopril  Unnecessary to be on two anti-emetics and both
 Multiple drugs with antihypertensive effects may are dopamine antagonists that cross the BBB
result in hypotension. and will worsen her parkinsonian symptoms. Stop
 There is no specific COPD therapy. them both and use domperidone to control her
nausea.
 If her psychiatric symptoms persist after the dose
st
of co-careldopa has been reduced, 1 line
treatment would be an atypical anti-psychotic,
such as quetiapine.

© Alasdair Scott, 2012 89


Useful Drugs to Know

Drug Dose
Amoxicillin 500mg TDS PO
Clarithromycin 500mg BD PO
Trimethoprim 200mg BD PO
Co-amoxiclav 1.2g TDS IV
Simvastatin 20mg OD nocte PO
Nifedipine MR 20mg OD PO
Lisinopril HTN: 10mg OD PO
HF: 2.5mg OD PO
Bisoprolol HF: 1.25mg OD PO
Paracetamol 1g QDS PO
Codeine Phosphate 30mg every 4h PRN PO
Max 240mg daily
Tramadol 50mg every 4h PRN PO
Max 300mg daily
Enoxaparin Treatment: 1.5mg/kg/24h SC
Prophylaxis: 40mg OD SC

© Alasdair Scott, 2012 90

You might also like